Conference Notes 7-17-2012

CONFERENCE NOTES 7-17-2012

MOTZNY   EMS STUDY GUIDE

Triage colors: green = walking wounded, red=emergent, yellow=care can be delayed for a few hours, black=dead.

JCAHO requires a hospital disaster plan to be activated 2 times per year.

A disaster is defined when an event overwhelms a facility’s ability to respond appropriately.

The most common important problem in a disaster is communication difficulties.

Mass gathering =1000 or more people at an event.  The most common injuryat a mass gathering is dermal injury.

Triage is a dynamic process that requires re-eval and re-assessment of patients.

The command station at a HAZMAT scene should be uphill and upwind from the hazardous materials.

Regarding decontamination; if there is ocular exposure address eye irrigation first.  Gross decontamination takes precedence over airway management.  Removing clothes is the first and usually most effective step of decontamination.

Class A bioterrorism agent:  Anthrax, plague, ebola, small pox, botulism, tularemia.     

Radiation:  Alpha particles can be blocked by paper or clothing.  It is unlikely to give whole body radiation unless ingested.   The earliest lab indicator of acute radiation syndrome is decreased lymphocyte count.  

Under NIMS, in a disaster situation, materials management is under the responsibility of the Logistics Section of the disaster response team.  Logistics is in charge of all the equipement and materials necessary for a disaster response.

START triage algorithms are based on a quick assessment of the patients respirations, perfusion and mental status.  

TOERNE    ACETAMINOPHEN AND SALICYLATE TOXICTY

Acetaminophen (APAP):  Completely absorbed within 4 hours.   A 2 hour Tylenol level predicts a 4 hour level.   APAP is eliminated by glucuronidation and sulfation.  5-10% is metabolized by the cytochromes to NAPQI which is the toxic metabolite.   NAPQI attacks cells in the liver.  Glutathione detox’s normally .  But if glutathione gets depleted in a Tylenol overdose you get liver damage.   Extended relief acetaminophen tab has similar overdose characteristics as regular tab.  If your 4 hour level is non-toxic then your 6 and 8 hours levels will likely be non-toxic as well.  Still, check 6 and 8 hour levels.

APAP overdose is initially asymptomatic.  After 24 hours pt’s develop nausea and vomiting.   Time of ingestion is the earliest time, not the latest time of ingestion.  

No routine gastric lavage or activated charcoal for APAP overdoses.  

Toxic ingestion of APAP=150mg/l at 4 hours  to 4mg/l at 24 hours.

N-acetylcystine (NAC) is the antidote for Tylenol overdose.  Oral dosing is  140mg/kg initial dose  followed by 70mg/kg q 4 hours for 72 hours.   Now IV Nac is available with a 21 hour protocol.  You can probably stop NAC when serum Tylenol level is less than 10mg/l.   NAC can falsely elevate the INR slightly.   If patient has signs of liver damage, you give NAC until liver enzymes are less than 1000.  Ted recommends using IV acetadote over po NAC.   It is just easier.  

Liver transplant criteria: Elevated INR, elevated Cr, and encephalopathy.    To increase sensitivity to pick up all potential transplant patients is lactate >3.5 or abg <7.3 after initial resuscitation.    You will see these abnormalities in late presenting patients or patients in the ICU who are not doing well.   If a patient with acidosis or diminished liver function, transfer that patient to a transplant center.

Salicylate (ASA):  Oil of wintergreen has a massive amount of salicylate in it’s formulation.   ICY-HOT also has salicilyate in it’s formulation.   

Treat with IV fluids, Bicarb drip and repeated doses of activated charcoal.   You want to alkalinize the serum and urine to ion trap the ASA in the urine.   This mechanism also keeps hydrogen ions out of the cns and heart.   You need to supplement potassium as well.

ASA  uncouples oxidative phosphorylation.   Most common cause of death is cardiac dysfunction.   Pt’s will have tinnitus.   Pt’s will have tachypnea due to metabolic acidosis and primary respiratory alkalosis.

Chronic ASA is very problematic due to whole body burden of ASA.

Harwood Comment: Is there vomiting with oral potassium supplementation in these patients.  Ted replied he has not seen that problem.

Indications for dialysis: deterioration, aletered mental status, pulmonary edema, severe acidosis, renal failure, worsening coagulopathy, ASA concentrations of 80-100 mg/dl.

Ted made a comment that there are case reports of ASA toxicity developing in a patient with an initial neg level due to irregular absorption of ASA.

 

 

VILLANO   SNAKE ENVENOMATION

Red on yellow, Kill a fellow=Coral snake.   Red on black, venom lack=non-venomous king snake.  Coral snakes are common in the southern US.

Crotalids (Rattlesnakes and others) have elliptical eyes, triangular head, a heat sensing pit, and retractible fangs.

Snake venom: crotalids have proteolytic venoms and can cause systemic toxicity.  Elapid venoms are neuro toxins.

Crotalid bites cause local reactions.  Can have systemic symptoms as well including hematologic and gi symptoms, rhabdomyolysis.

Elapid bites (coral snakes): pt can be asymptomatic up to 12 hours then develop paralysis.

Poison control center # anywhere in US 1-800-222-1222.

Asymptomatic patients: crotalid bites observe for 8 hours.   Elapid bites give antivenin.

Field management: ok to irrigate and splint wound and avoid strenuous exertion, and take a picture if snake if easily available.  Don’t use tourniquets, suction, direct handling  of snake.

Management: Antivenom, tetanus prophylaxis, avoid blood products unless pt is hemorrhaging to death.  Antivenom should be used prior to fasciotomy for swollen limb.   Anti-venom is not a weight based dose.  Same dose for adults and kids.   Patients may have an anaphylactoid reaction from antivenom.   4-6 doses of Crofab antivenom given for spread of swelling, hematologic abnormalities, or unstable vital signs.  (spread-bled-almost dead)

Elapid (coral snakes) bites management: look for signs of weakness including respiratory parameters such as low maximal inspiratory pressure, capnography.  Everybody gets coral snake antivenin because pt’s can be assymptomatic initially.  Repeat dosing based on clinical status.  This antivenin can cause anaphylatoid reactions as well.

Non-venomous snakebites: get xray to check for fb. Give prophylactic antibiotics (gram positive coverage). 

Gila monster bite: wound care, antibiotic prophylaxis. No anti-venin.

Harwood comment: for the snake mnemonic start with red like a fang mark.  Red on yellow, kill a fellow. Red on black, venom lack.    

KATIYAR  MARINE ENVENOMATIONS

Jelly fish have nematocysts with venom that is painful and toxic to humans.  Nematocysts can be rinsed off with vinegar.

Cubozoa: Box Jellyfish is indigenous to northern Australia and SE asia. Clear color. Can cause local reaction and in worst case cardiovascular collapse. Rinse off with vinegar.  Don’t rinse with fresh water or urine.  Nematocysts need to be removed afterward with tweezers or razor.  Give antivenom 1-2 amps IV or 3 amps IM.  It is sheep derived.  Can cause anaphylaxis or serum sickness.

Irukandji Jelly fish:  Irukandji syndrome=30 minutes after stung, low back pain, muscle cramps, hair stands up, anxiety, sweating, tachy, vomit, oliguria, cerebral edema, pulmonary edema.   Lasts 5 days to 2 weeks.

Portuguese man of war: Larger than box jelly fish, Blue color. Has blue sail above water. Tentacles may be 100 feet long. Complex venom that is hemolytic and cytolytic.  Intense local pain, hemolysis, cardiac conduction abnormality, recurrent urticaria.   TX: neutralize with salt water.  Don’t use vinegar because it causes 30% nematocyst discharge.

Seabathers eruption: Due to larvae of jelly fish.  Contact dermatitis.  Pruritic papules.  Lasts for 2 weeks.  TX with steroids and antihistamines.

Stingray: Has a serrated spine with venom. If you step on them they can sting you.  Steve Irwin was killed by one.  Local reaction, tremors, convulsions, CV collapse.  Tx with tetanus shot, hot water deactives venom (soak extremity), irrigate wound, systemic analgesia, cover gram negs with abx.

Lionfish:  Envenomation causes pain, swelling, blistering, weaknss.  Treat with hot water and wound care, and tetanus shot.

Stonefish: Highly toxic.  Immediate and intense pain, delirium and cardiovascular collapse.   Treat with hot water, support vitals, wound care, and tetanus shot, antibiotics.

Sea Urchins: pain and pruritus.  Relatively benign envenomations.  Treat with hot water.  Get xray to check for retained spine.  Treat pain.

NIKKI NINO   MD DOCUMENTATION

Laceration repair is the most common missing piece of documentation.

For all procedures use the procedure macros in Picis.  

Conference Notes 7-3-2012

Conference Notes 7-3-2012

Chandra/Harwood   Oral Boards

Case 1.  Multiple Blunt Trauma with left hemothorax, splenic injury and epidural hematoma.   Management  required  intubation, left chest tube, identify epidural hematoma/splenic injury. Pt needs to go to OR emergently.

Harwood comment: You can identify epidural hematoma (between dura and skull) by thinking high pressure causes bulging toward the midline.  Low pressure subdural (between dura and arachnoid)does not cause a bulge toward the midline.

Case 2.  Syncope due to PE/FX toe.   Management requires identify CT, give heparin, reduce toe fx.

Harwood comment: S1Q3T3 on EKG and Westermark sign (unilateral oligemia due to clot) on CXR were present in this case.

Case3.  Maisonneuve Fx.  Identify fracture pattern, splint, urgent ortho consult.  Surgery not required emergently but in a prompt fashion.

Asokan  Emergency CXR Evaluation

If patient has hx of a fall, a radiologist looks for pneumothorax, pleural fluid, rib fx’s, or vascular injury.

If the trachea deviates toward a soft tissue density, the soft tissue density is likely not a mass but some scarring or buckling.

If a patient has a widened mediastinum due to a dissection, it is because of a mediastinal hematoma.  The dissection itself does not cause a widened mediastinum.

Indications for surgery in aneurysm is 6cm in the thoracic aorta and 5cm in the abdominal aorta.

Superior mediastinal widening  has a differential of 4T’s: thymoma, teratoma, thyroid, terrible lymphoma.

On lateral view: Posterior infiltrates are in either right or left lower lobe. Anterior infiltrates are in the RML or left lingula.  On anterior view: RLL infiltrates should basically be on the diaphragm.  RML infiltrate may obscure the right heart border. RUL infiltrate should be at apex.

Hemithorax with whiteout : if trachea deviates to side of white out consider collapse or pneumonectomy.  If trachea deviates away consider hemothorax/mass/effusion.

CHASTAIN  SIGN OUT

Transfer of information AND responsibility for the patient.

IPASS: illness severity, patient summary, action list, situation awareness, and synthesis.

It is important for the receiving team to ask questions about the case.

Signed out patients need to have a complete H and P note written by the leaving team.

WILLISON  EKG

Osborne waves  indicate hypothermia

WPW: accessory tract can lead to SVT’s of different types.  Antidromic conduction has a wide QRS and requires procainamide or cardioversion.

V1 if RBBB pattern: Taller left rabbit ear suggests ventricular tachycardia.

If you find non-specific st changes in a anatomic distribution, get serial ekg’s to look for st elevation.

AVR with tall/wide terminal r wave think: TCA, or Benadryl or cocaine.

Cerebral T waves: huge deep inverted t waves due to acute intracranial hemorrhage.

Wellens: biphasic t wave in V3 suggests severe proximal LAD lesion.  High risk for v-fib on treadmill.

Brugada: Saddleback t wave changes in septal leads.  Associated with sudden death.  Needs AICD.

WISE HEMOPTYSIS 

Gas exchange impaired with >400ml of blood in alveolar space.

Mild hemoptysis without risk factors: CXR and outpt f/u

Causes: bronchitis, pneumonia, abscess, tb, lung carcinoma, pe, behcet, goodpastures, bronchiectasis, warfarin, crack lung, bioterrorism-agents .  Top 3 in US: bronchitis, bronchiectasis, cancer, pneumonia.  Tops in world: TB

Earlier bronchoscopy= higher yield.  CT may be useful in massive hemoptysis. CT plus bronch identified source in 93% of patients.

Lateral decubitus position with bleeding lung on downside may protect the good lung.  Selective mainstem intubation or double lumen ET tube may also be effective. 

Consider FEIBA for the massive hemoptysis pt on warfarin or other anticoagulant.

Use an 8FR ETT when intubating to allow for bronchoscope.

 Barounis comment: Dr. Hanif said you can pass a pediatric foley through the ET tube and pass the foley into right mainstem bronchus and inflate balloon to occlude right bronchus.   

KULSTAD/TEKWANI/WATTS    STATS

High specificity tests rule in disease. SPIN=specificity, positive results, rules in the diagnosis.

High sensitivity tests rule out disease. SNOUT=sensitivity, negative result, rules out the diagnosis.

Positive predictive values is highly dependent on prevalence rate of a disease. PPV can change despite no change in the sensitivity or specificity of a test based only on  the different prevalances.

Bayes’ theorem: new info should be interpreted in light of what is already known.  You need to consider the pre-test probability of disease.  Can base on the literature or clinical gestault.

Positive Liklihood ratio: prob of + test in presence of dz/prob of + test in absence of disease.  If LR is >1 the result is more likely to be positive in a pt with disease than without disease.  The benchmark for A very good  LR+ is 10.  The benchmark for a very good LR- is  0.1.

Harwood Comment: Determining the pretest probability is key to determining an accurate post test probability.   

Heart Score for Chest Pain:

History

Highly suspicious

2

Moderately suspicious

1

Slightly suspicious

0

 

   

ECG

Significant ST-depression

2

Non specific repolarisation disturbance

1

Normal

0

 

   

Age

≥ 65 years

2

45 – 65 years

1

≤ 45 years

0

 

   

Risk Factors

≥ 3 risk factors or history of atherosclerotic disease

2

1 or 2 risk factors

1

No risk factors known

0

 

   

Troponin

≥ 3x normal limit

2

1 – 3x normal limit

1

≤ normal limit

0

 

HEART Score

Risk of MACE

Proposed Policy

0 - 3

0,9%

Discharge

4 - 6

12%

STRESS

7 - 10

65%

ANGIO

 

We then discussed 8 cases in small groups developing post test probabilities for strep testing, d-dimer, ct for appy, and dopplers for dvt.  

 

JAKUBOWITZ   CHF/ASTHMA

Observation status delivers equivalent clinical care to admission at a lower cost.

Asthma Protocol: Bread and butter asthma patient not better in 3 hours should go into this protocol.  DC home with albuterol/oral steroids/inhaled steroids/asthma action plan/follow up.

CHF Protocol:  Patients with new CHF, abnormal labs, unstable vitals, o2sat <90% are excluded.

Less than 10 patients last year were admitted into each of the CHF or Asthma Protocols.

Observation management may be more expensive for patient than inpatient management.  However, some of these protocol patients may not meet criteria for admission.

 

 

Conference Notes 6-12-2012

Conference Notes 6-12-2012

JOINT PEDS/EM CONFERENCE NEONATAL EMERGENCIES

Case #1:  Hypoplastic Left Heart Syndrome

MISFIT mnemonic : Metabolic, Inborn Errors, Seizures, Formula problems (hypernatremia/hyponatremia), Intestinal disasters, Toxins, Sepsis.  

If neonate presents with hx of fever but afebrile in ER.  Panel recommended partial septic workup with CBC, blood culture, and urine dip and observation period with recheck of temp.  If you admit for obs, if you want to give antibiotics, you have to do LP.  

In sick neonate you want to get an ABG.   

Go rapidly to IO line if you have difficulty getting IV access in a neonate.  Bolus IV fluids in rapid fashion over about  5 minutes.  You may have to give by push syringe thru iv/io.

Hyper-oxygenation test is not reliable.  It can give misleading results and increase pulmonary flow and worsen patient’s condition.  Similarly the BNP test has no utility in this age group.  Very ill hypoplasts have normal BNP’s.

Agreement among panel that if patient is unstable don’t do lp prior to antibiotics.  But Dr. Collins pleaded to get at least a blood culture prior to antibiotics.

Dr. Roy: If child is grey color (poorly perfused)not cyanotic you have to work up sepsis/cardiac/in-born errors/non-accidental trauma.  

Panel: nightmare baby is shocky baby with low normal O2 sat.  Gotta treat sepsis and at the same time work up cardiac and in-born errors and non-accident trauma.  Panelists suggested giving both antibiotics and prostaglandin if you can’t get a rapid echo and need to transfer patient.

Pre and Post Ductal Pulse Ox measurement (pulse ox on right hand and either foot) can be a sign of ductal dependent congenital heart disease.    Could be used in ER.  If one measurement is less than 90% or is significantly lower than the other measurement the test is abnormal.

Too much oxygen in these kids is more dangerous than too much fluids.  20cc/kg bolus was considered safe  by all panelists.

Case #2:   Evaluate for sepsis first because it is most common,  congenital heart dz and inborn errors are much less common.   Again, ABG was touted as a useful test by the panel in the undifferentiated ill neonate.   Panel felt Ammonia level was not a test to be getting in all these kids until you have considered other diagnoses.  Ammonia levels can be unreliable in the acutely ill child.  

Sam Lam: Inborn error of metabolism kids look sick, vomiting, shocky.  Labs will show hypoglycemia and acidosis.     Dr. Collins pleads again for blood cultures and antibiotics in this kid because sepsis is still the most common diagnosis.   If you do an LP the most important test is the culture.   If you have enough fluid then get cell count, gram stain, protein, and glucose.   Extra fluid can be saved for HSV pcr later if indicated.

 

Lovell chiming in to emphasize my favorite pearls from joint Peds/EM conference:

Common things common:  r/o sepsis, peds cards problems before worrying about inborn errors of metabolism (really rare) in a sick neonate-think about and treat both.  Get the blood culture before antibiotics, but defer the LP in shocky kids-stress of LP can make them worse.  

Hyperoxia test probably not helpful due to mixed lesions being able to mount reasonable sats, and putting cyanotic heart lesion kids on high flow oxygen can make them worse (adversely affect perfusion).  Instead, think about doing a O2 sat on right arm and either leg (pre/post ductal) to look for discrepancy and pick up ductal lesions.

 

 

DAVE CUMMINS    MY FIRST YEAR OUT  AFTER RESIDENCY

Dave discovered that single coverage ER’s can be a lonely place for a doc.

People don’t really question you that much as an attending.

Pacemaker mediated tachycardia is best treated with a magnet or pacemaker computer rather than cardioverting.

Not every ED operates the same as ACMC.   They may manage afib and other problems differently than we do.

Dave had to treat a case of trachea-inominate fistula bleeding.  This is frequently a lethal complication of tracheostomy.

From Robert’s Clinical Procedures in EM Text: Control of innominate artery bleeding by digital pressure. Be aware that minor bleeding may be a sentinel event, and a harbinger of a subsequent major hemorrhage. When major bleeding occurs and a cuffed tracheostomy tube is present, overinflation of the tube cuff may temporize (see text). When this is unsuccessful or a cuffed tube is not available, use the illustrated maneuver; digital pressure should be applied to the anterior tracheal wall through the tracheostomy. The index finger is placed within the trachea and then pulled against the anterior tracheal wall, allowing the airway to remain partially open. The artery is compressed between the index finger and the thumb—placed over the neck. Digital compression of the innominate artery is a temporizing procedure, until definitive (operative management) of the bleed is obtained.

Success comes mostly from the relationships you build with others.  It has much less to do with how good you are.

Dave’s Top Suggestions for year 1

  1. Be nice
  2. Never yell
  3. Mirror the behavior of your favorite attending
  4. Be decisive and have a plan
  5. Don’t take yourself/status too seriously
  6. Learn everyone’s name quickly
  7. Work less than in Residency
  8. Spoil yourself then SAVE ALOT!!!!!
  9. Get a good financial planner
  10. Develop another work interest.
  11. Be generous with your time/money/espertise
  12. Don’t prescribe  narcs or benzos to friends/family

ERIN ZIMNY   PALLIATIVE CARE

Palliative care is symptom management, communication, and coordination of care.

ED is critical setting for pt’s with cancer and other terminal illnesses because we initiate the trajectory of care.

Patients with cancer or other chronic illnesses: 94% have physical symptoms, 72% have financial concerns.

Goal: Cancer patients go to ER only once in last 30 days of life.

Palliative Care does the right thing for the patient and also saves $.

Case #1: Hospice Patient in the ER.  They may come for increased symptom,  new problem, self referral for stress/inability to cope.

New care model for advanced cancer is a relative gradient of cancer therapy and palliative care.  This is a change from the past which was cancer treatment alone until it was determined unsuccessful then palliative care.

Hospice Myths:  Pt’s have to be DNR to be in hospice.   Hospice does not treat infections.  Hospice withholds parenteral nutrition.    Hospice has to be revoked on arrival to ER.

ER staff should call hospice staff.  Identify trigger for ED visit.   If deterioration is imminent discuss with pt and family.   Give family/patient emotional support.    Dispo can be home, hospice, revoke hospice.

IV decadron makes cancer patients feel a lot better.  It relieves symptoms from tumor burden.  It will also increase appetite.

Treatment of Nausea: There is more than zofran.   Benzos, antihistamines, raglan, compazine, haldol 2-5mg IV (blocks the chemoreceptor trigger zone), droperadole is another option.

Treatment of Constipation: Use a combination of a stimulant and a stool softner.   Senna and colace is a good combination for constipation prophylaxis. Harwood recommends Pericolace as a combo med with both senna/colace.    Methylnaltrexone blocks opioid receptors in gut.  Works like a miracle but is expensive.     If patient comes to ER with constipation Erin recommends  giving lactulose from above and ducolax from below.

 

Case #2: Dementia Patient

Dementia is a progressive terminal disease which is irreversible.  There are identifiable stages.  Stage 7c with loss of ambulation and not speaking, they have less than 6 months to live.

It is ok, if in your medical judgment the patient is going to die soon, to decide for the family that the patient is DNR.   If they assent to this decision, you make the patient DNR.  

Delivering Bad News:  Advance prep, build a relationship, communicate well, deal with reactions, encourage and validate emotions.      Find out what they know, be frank but compassionate, allow silence/tears,  summarize and repeat info, encourage questions.   

Things not to say: I understand how you feel.  It could be worse. Nothing more can be done. We all die. Avoid euphemisms, use the word death/dead.

Case #3:  Patient with metastatic lung cancer in extremis.   Husband screaming “do something!”

Erin stabilized patient with intubation and pressors.   Then had long discussion with husband.  Daughter comes and says “mom never wanted to be on a ventilator”.  It was decided to extubate patient in ER.  Dyspnea treated with morphine.

Treat dyspnea with: oxygen, fans may be effective, morphine 2-5mg Q15-30 min (this is symptom treatment not euthanasia), ativan 0.5, humidiity , elevate head of bed, educate family, treat secretions with atropine.

Oral morphine 15 mg is a reasonable starting dose q4 hours for cancer patient failing norco.

SubQ dosing of opioids is less painful than IM dosing.

Don’t start a patient new on a fentanyl patch in the ED.  You can increase the patch dose for someone already using a fentanyl patch.

ACMC has a Palliative Care Team 684-8117.  Dr. Kozyckyj and Lynn Sevic, RN. 

SINNOTT     SENIOR PEARLS 

Conference Notes 6-5-2012

Conference Notes 6-5-2012

STRASBURGER     PSYCHIATRIC STUDY GUIDE

Droperidol and haldol can prolong the QT interval and precipitate torsades.   Check an EKG prior to giving either drug.

Treat dystonic reaction with benztropine (cogentin) and Benadryl.

Harwood comment: sometimes Benadryl doesn’t work in that case give cogentin and a few for home to prevent recurrence.

Low potency antipsychotics have more sedation and hypotension and anticholinergic symptoms

High potency antipsychotics have more dystonia and tardive dyskinesia.

Harwood comment: It’s ridiculous that there is a question about low potency  anti-psych drugs because they aren’t used anymore.

Harwood comment: Most people with panic attacks will not have a persistent resting tachycardia.  If pt is persistently tachycardic, check thyroid studies and consider other diagnoses.

3% of patients with steroid psychosis commit suicide.

Conversion disorder:  symptom that is a change or loss of function. Recent stressor, symptom not explained by any organic disease.  Labelle indifference.

Harwood comment:  Consult appropriate specialist to eval patient prior to making diagnosis of conversion disorder and discharging patient.

Munchhausen by proxy: caregiver fabricates illness in those who are in their care.   Form of child abuse.

SADPERSONS: mnemonic for suicidal risk.  Sex, age, depression/hopelessness, previous, excessive etoh, rational thinking loss, separated, organized attempt, no social support, stated future attempt.   Hopelessness, loss of rational thinking, stated future attempt all get 2 points and are higher risk than the other factors which get 1 point each.

TCA overdose: look for wide QRS and tall R wave in AVR on EKG.

Effexor (venlafaxine) has sodium channel blocking effects and will have similar EKG findings to TCA’s.

Discussion on neuroleptic malignant syndrome among attending: cool patient, use benzos, consult tox/neuro, give dantrolene.   

COLLANDER    5 SLIDE F/U

Patient with purple urine in foley bag.   Urine culture was positive for proteus and e coli.  Both organisms susceptible to cipro.

Purple urine bag syndrome: tryptophan metabolized to indole.  Liver converts indole to indoxyl sulfate.  Bacteria convert indoxyl sulfate to indirubin and indigo.   Usually in asymptomatic patients with chronic foley catheters.  Constipation predisposes to more uptake of tryptophan so more likely to have purple urine.   Multiple gram neg organisms can cause this.   Treatment is change out the catheter and burine bag and treat uti.  You can use the usual urine antibiotics.

Child with blue urine is likely to be due to food coloring or familial hypercalcemia.  Check a calcium level.

RICCARDI    M AND M

75 YO male, multiple chronic illnesses, hx of cholangitis. Presents febrile and tachy with green biliary drainage.    Antibiotics started.

Cholangitis: bile stasis that develops infection.   High rate of sepsis.  Stones can act as a nidus for infection.   Treatment is biliary drainage and antibiotics.

Charcot’s triad; fever, pain, jaundice.  Reynold’s pentad is the addition of hypotension and altered mental status.

80% will respond to antibiotics.  20% require emergent decompression of biliary blockage.

Patient deteriorated in ER.  Developed an urticarial rash.   Pt was treated successfully for anaphylaxis.  It was later learned pt has a pcn allergy.

Anaphylaxis criteria  there are three: 1. Acute onset of skin and respiratory or low bp effects. Or  2.  Any 2 of skin, gi, respiratory, bp  effects after allergen. or 3. Low bp after exposure to known allergen.

Treat with epinephrine 0.01mg/kg    in thigh.

Biphasic reactions are possible up to 72 hours out.   Patients should be observed for 4-6 hours in ER.

Pt had antibiotics changed, went for ERCP and eventually was dischared from hospital.

Dr. Riccardi discussed the importance of being alert to anchor bias.  Just because the patient is triaged to a lower acuity area of the ED does not preclude they may have a serious, or emergent illness or injury.   Elderly are more prone to under-triage.

Dr. Riccardi also discussed confirmation bias.  The EP is suspicious of a certain diagnosis and uses data collected to confirm this bias.   The EP tends to discount non-confirmatory data or not pursue data that would lead to another diagnosis.

Beyesian reasoning may help fight confirmation bias.  Does my test results raise or lower my pretest probability?

C. Kulstad comment: Be alert for under-triaged nursing home patients in the hallway.

Joan Coghlan  comment: Ask yourself, if I saw this patient in the Critical Care room would I do the same thing?   Frequent ED users also require extra vigilance.

McDERMOTT AND LAMBERT     ORAL BOARDS

 Case 1.   Pregnant patient with adominal pain.  Distended bladder.  Incarcerated uterus. Rarely  uterus can’t come out of the pelvis at about 10-12 weeks and will block bladder drainage.   

Case2.  Lower extremity compartment syndrome.     Severe calf pain.  Use Stryker to measure compartment pressure.   Normal pressure is 0-5.   Admit for pressure over 20.  Surgery for pressure over 40 or within 20 of diastolic blood pressure.

Case3. Optic Neuritis due to Multiple Sclerosis.  Treatment with IV Solumedrol.  Fundus will be normal.  Pt will have an afferent papillary defect.   Pts may have eye pain.

Joan Coghlan comment: Pain out of proportion should make you think either ischemia, compartment syndrome, phlegmasia, tendon rupture.     Know the likely disease entities at various  gestational ages.  Ectopic at 6-8 weeks, uterine entrapment at 10-12 weeks,  second trimester is appy or cholecystectomy, third trimester is abruption and previa.

CALLAHAN  FOSTERING RESILIENCE

Resilience: the ability to bounce back and endure adversity during residency.

Not resilient resident:

Resilient resident:

Hero’s Journey: Described in 1949 by Joseph Kimble.  Start with known (home field) and travel to unknown world (call to adventure), work thru challenges and temptations, reach an abyss (doubt of success), revelation, transformation, atonement, success and return to the known (home field).

Nutrition, aerobic exercise, sleep lead to healthy living but do not guarantee resilience.

Lessons from Athletics: How you think about failure whether it is permanent of fleeting is a part of resilience.  2nd category is locus of control.  If a person takes more internal control of their performance and not blame external factors they are more likely to be resilient.  3rd category is a sense of hope/optimism/higher purpose.

You have to dispute negative thoughts.  Either do it yourself or have a mentor to help you do that.

Decatastrophize problems.    Consider worst case scenario, consider best case scenario, and settle on most likely scenario.

Lessons from the military: NAVY SEAL’s are considered the most resilient people on the planet.  Optimism, perseverance, responsibility, integrity, support each other during training, self-efficacy (they believe they control their destiny), earned.

Comprehensive Soldier Fitness: Family, physical, social, emotional, and spiritual facets.  Military doesn’t have time to wait because currently there are more military deaths from suicide than from combat.

Marty Seligman: Formula for resilience or positive thinking: Content with the past, happy in the present, hopeful for the future.    Authentichappiness.com.   Dr. Seligman runs the Comprehensive Soldier Fitness program.

Not much data to support specific resilience training.   3 factors do have some support: positive thinking, positive affect, positive coping.   2 other factors also important: realism and behavior control.

Positive coping, support from family/resident class, positive climate in Unit/program, Belongingness in Community/hospital.    Physical fitness is not a link to resilience.

Lessons from Medical field: SMART program at Mayo: Attention wanders to threats/pleasure/novelty.  Spend a lot of brain power in the past and future.   We should switch to the present moment.   Gotta override the limbic system.  Gotta train the mind to quiet the limbic center.   Focus on the current moment.  Interpret life with more flexibility, gratitude, compassion, forgiveness, and higher meaning.

Conference Notes 5-29-2012

Conference Notes 5-29-2012

GRIPPO-FELDER ORAL BOARDS

Case 1:  Borhaave’s Esophogus.    Consider PE, consult surgery, start broad spectrum antibiotics.    Pregnancy and ETOH are risk factors for esophageal rupture.   Don’t do a barium swallow because  arium is not good for the mediastinum.  Gastrograffen is preferred to evaluate with a swallow study or chest CT.

Harwood comment: If you see mediastinal air or air in soft tissue either on CXR or CT with this symptom complex, just consult surgery and start antibiotics.   The ER doc doesn’t need to order an esophogram.

Case 2: Lyme Disease.  Recognize erythema chronicum migrans, treat with doxy  for kids over 8 and non-pregnant.  21 day course of doxy, amoxicillin, cefuroxime, or macrolide.   Tick has to be on the patient for 36 hours to transmit disease.

Harwood comment:  For oral exam with non-critical/toxic patient you don’t have to put an IV in. You may get scored down for system based practice.  Think out loud so the examiner knows what you know.

Case 3: Polytrauma.  Cspine immobilization, secure airway, chest tube, re-assess vital signs.   Always remember to get complete set of vitals including accucheck glucose and UCG.  Ask for family/paramedics. Treat pain. Be systematic in your exam for injuries.

BOLTON   FUTILITY

Decisions: Respect the patient’s autonomy.  The decision should be impartial and follow the Golden Rule (treat others as you would want to be treated) or Platinum Rule (treat other as they would want to be treated).   Universable=categorical imperative=moral act that is always right in the same situation.  Interpersonal justification=would you be comfortable with your decision if it is written in the newspaper?   

Futility= action that has no useful purpose.   AMA CEJA: there is no accepted definition of medical futility.

80% of persons die in a medical environment.  People prioritize quality of life, touch of family at the time of death.  They don’t prioritize prolongation of life at any cost.

Futility problem: patients and families have unreasonable expectations of the capacity of medical care to return the patient to prior state of health.   This is based on peoples exposure to TV and movies  where CPR and ICU care provide miracle saves.   Physicians have a lot of difficulty prognosticating to patients and family with the goal of lower expectations.   We frequently abandon patients/families to their own autonomy. (Do what you think is best).

We should help families/patients understand their goals prior to making end of life decisions.

You don’t have to be a DNR patient to be in hospice.  It is however somewhat contradictory philosophically.  The criteria for hospice is only an expected life span of less than 6 months.

Pt’s don’t have the right to demand treatment.  Beneficence: CPR has 0% chance of survival with metastatic neoplasm admitted to the hospital.  Justice: Fair resource allocation.

Harwood comment:  After every successful resuscitation, look for the underlying cancer.

Girzadas comment:  The EP can also factor in the pain or  lawsuit  that may  impact the treating physician from the family.

Barounis comment:   Recent case in ER having to decide whether to give post-resuscitation hypothermia in a young patient with metatstatic cancer who had ROSC following CPR. Some ICU personnel complained that the ER staff should not have cooled the patient.    Everyone agreed that the case was extraordinarily difficult.   Most agreed that if pt was not DNR and resuscitation was done and had ROSC then  you have to give hypothermia therapy.    

Willison comment:  The LET form is very imperfect.   Most people at the lecture agreed that the multiple check box format brings up ethical inconsistencies.

WILLISON/SALZMAN  TRAUMA RESUSCITATION

Be sure to use personal protection like masks/eye shields/gloves/gowns/shoe covers.

Try to organize your team as much as possible.

Abnormal vitals, pregnant patients, elderly, another fatality, fall over 20 feet, auto vs. ped are all prearrival markers for badness.

To intubate, loesen c-collar but have a second person hold in line stabilization until tube is secured.  Then re-apply the collar.

New info:  28 or 32 FR tube was no better or worse than 38 or 40FR tube.

Don’t  “rock the pelvis”  just give one firm push on the ASIS bilat. If it moves it is fractured.

Fix scalp lacs. Patients can bleed severely and even  rarely bleed to death.  Tourniquets (BP cuff at 300mm hg) can sometime be life saving but use these rarely and cautiously.

When transfusing large volume of prbc’s, match units of prbc’s, ffp and possibly platelets.

Positive FAST scan, Systolic <90, HR>120, penetrating injuries: 3-4 of these are high risk for needing massive transfusion.  1 probably not.

Penetrating wounds to abdomen/flank/low back/pelvis require rectal exam.  Other injuries you can be more selective with rectal exams.

Keep patients warm in ER.

New Thoughts from Scott Weingart for Traumatic Arrest: No closed chest CPR, no acls meds, first airway is LMA, bilat finger thoracostomy, cardiac ultrasound looking for tamponade.

Salzman comments: ED thoracotomy only good for penetrating chest wounds (optimally stab wound to heart) that cause tamponade.  Heart is very delicate and it can be easily damaged by a scalpel or by fingers during internal CPR.   ED thoracotomy  for blunt trauma is futile.

Can consider ED thoracotomy to cross clamp aorta for a patient who has exsanguinated from a limb amputation.  Give patient rapid prbc transfusion and do cardiac massage.

Harwood comment: It’s a thought.

Barounis comment/Salzman response: Massive transfusion protocol for untable, hypotensive pelvic fractures should be started in ED.      Pt’s with other types of injury, the protocol  has to be used more judiciously.   If the protocol is initiated you can always back off if bleeding lessens.  Patients that need to go to IR are more likely to need the protocol than patients going directly to OR.   Level 1 transfuser is a critical tool during resuscitation.

FORT   5 SLIDE F/U

Poly drug overdose including TCA.   PT was agitated.   Intubated, sedated with propofol.   Toxsicon recommended charcoal, serial ekg’s and bicarb for QRS >100ms.   Initial ekg was ok/not wide/no terminal prolongation of QRS (big R wave) in AVR.   Remained stable over 16 hours in ER.   Was extubated and transferred to psychiatry.

TCA: sodium channel block, antihistamine,  anti-muscarinic, K efflux blockade, alpha blocker, and gaba blockade effects.  “Dirty Drug”   has multiple effects.   

Treatment: Seizures give Benzos.   Hypotension give fluids and pressors.   QRS prolongation give sodium bicarb.   Dialysis is worthless due to high volume of distribution.

Harwood comment: Bicarb is the main treatment for EKG abnormalities.   Brian said if bicarb not helping  consider magnesium.

HERRMANN  5 SLIDE F/U

Child with GSW to right thigh.  Pt had right femoral arterial injury.  Hard signs with loss of distal pulses and pulsatile bleeding. Pt also had abnormal ABI’s.  CTA showed injury to femoral artery.   Prbc’s transfused.  Pt went OR.   Surgeons used saphenous vein from contralateral leg in reverse orientation (to negate the venous valves) to fix artery. 

ABI has 98% diagnostic accuracy.   Measure BP in all 4 extremities.   Divide ankle systolic BP by higher of two upper extremity systolic bp’s.   ABI<0.9 is abnormal and pt should get CTA or go to surgery.

Hard signs: abnormal pulse, arterial bleeding, pulsatile hematoma, bruit, thrill, distal ischemia.

Barounis comment: Vascular injury signs can wax and wane.  These patients are tricky.  Need re-exams if the initial decision is to observe so not to miss developing hard signs.

Chastain comment:  Compartment syndrome has been known to develop on trauma patients after the initial injury.  Stay alert even if you are tired.

KESSEN  RSI DRUGS

Pre-treatment: moderates reflexic sympathetic response to laryngoscopy.  Phayrnx and larynx are highly innervated with sympathetic and parasympathetic nerves.  LOAD:  Lido (no study shows neuro outcome improvement/Opioids (fentanyl  can be considered for pain) /Atropine (for kids <5yo getting succ)/

Nelson comment: Contrarian view is that all these pretreatment drugs increase complexity and delay intubation.

Etomidate is most hemodynamically neutral sedation drug.  Consider Ketamine as an alternative in the septic shock patient to avoid adrenal suppression.

Ketamine provides anesthesia and analgesia.   Increases cerebral blood flow.  May increase BP.  It is a bronchodilator.  May elevate ICP.   Watch out for emergence phenomenon.

Propofol causes anesthesia and amnesia.  May cause hypotension.  No analgesic properties.

Versed provides anesthesia/amnesia but not analgesia.   Can cause hypotension.

Barbiturates can provide anesthesia/amnesia and analgesia.   Hypotension.  Suppresses WBC function/recruitment.

Succinylcholine contraindicated in patients who have had  severe trauma,  burns, neuro injury  all more than 72 hours prior to ED visit.  These are not a problem if insult occurred the day of ED presentation.

Mistry comment:  Many absolute contraindications to succinylcholine are actually relative contraindications and succ is pretty safe.

Rocuronium has less than 1 minute onset. Intubation conditions are similar to succinylcholine.

Sugammadex is a reversal agent for rocuronium that is being tested in Europe.  Cuts the spontaneous ventillarion time from 400s to about 200s.

Harwood comment: Kid with severe astha, “Your risk of killing this patient is going up and up”    Use etomidate, atropine to decrease secretions, and succinylcholine if airway is not  predicted to be overly difficult.

 

Conference Notes 5-22-2012

Conference Notes 5-22-2012

BENJAMIN TICHO  OPHTHO TRAUMA

 20:20 vision means you see at 20 feet what a normal person is expected to see at 20 feet.  20:40 means you need to be at 20 feet to see what a normal person sees at 40 feet.   Basically think of it as the first 20 is the patient in comparison to the second number which is the normal person. 

CRAO have a generally poor outcome no matter what you do but you should still contact ophtho emergently.

The cornea has the most dense distribution of nerve endings in the entire body.

Dr. Ticho has seen severe injury to cornea requiring corneal transplant in patients who used topical anesthetic at home for corneal abrasion.   He advised strongly against giving topical anesthetic to patients with corneal abrasion.

Patching the eye for large corneal abrasions can speed healing.  Don’t patch more than 12 hours and the patch has to be tight enough to keep eyelid from opening.

Base injury to eye is worse than acid because base injury causes sapponification.   For both acid and base injury immediately irrigate the injured eye.   Get ph before and after irrigation.  Irrigate until ph gets between 7.3 to 7.7.

Treat superglue exposures to the eye with topical antibiotic ointment. 

KERWIN    STUDY GUIDE   IMAGING

Deep sulcus sign= pneumothorax

Delta sign=cerebral venous thrombosis.  Finding on posterior aspect, sagital sinus on CT brain.

On Chest xray, left mediastinal width greater than 5mm is a marker for aortic injury.  Left mediastinal width is measured from the spinous process to lateral border of aortic knob.   PAL CXR should be less than 5mm,   AP CXR the measurement should be less than 5.4mm

Bohler angle should be 20-40 degrees normally.   Less than 20 degrees suggests a fracture.   (memory hint: low score when bowling is bad.)

Hold patient’s metformin for 48 hours after they receive iv contrast to avoid metabolic acidosis.

Tram lines or train-track lines describe pneumatosis intestinalis and is indicative of NEC.

Thickened, non-compressible appendix of greater than 6mm in diameter is diagnostic for appendicitis. (memory hint: appendix is six)

Fluid in morrison’s pouch on ultrasound has pretty close to %100 positive predictive value for ruptured ectopic pregnancy.  

Scapho-lunate dissociation has a gap between the scaphoid and lunate called the Terry Thomas sign.

 Duodenal atresia, volvulus, annular pancreas are the differential diagnoses for the “double bubble sign”.

Cardiac standstill on echo during resuscitation has 100% PPV for death.

Can’t give gadolinium to pregnant patients because it crosses the placenta.  It is contraindicated, but there have not been reported fetal defects however.

Snowman sign of CXR in kids is a sign of Total Anamolous Pulmonary Return.  (memory hint: Frosty said he would “be back again some day”  that would definitely be an anamolous return)

 CT scan for PE in a pregnant patient has lower radiation dose to child than a VQ scan.  If you have to do VQ scan in a pregnant patient,  you can reduce radiation exposure to child by putting foley catheter in mom to remove radioactive urine.

MENON  M AND M

Obese=BMI of 30, Morbidly obese=BMI of 40, Super Obese=BMI 50

Obesity Hypoventilation Syndrome=Pickwickian syndrome.  BMI>30 PCo2>45 while awake, no other source of hypoventilation.  

When Intubating obese patients use  RAMP positioning.   Have the patient’s head elevated and face and jaw  parallel to the ceiling.  The patient’s external auditory meatus should also line up with their sternal notch.        

Pre-oxygenation increases the patient’s oxygen reservoir and denitrogenates the residual capacity of lungs.   3 minutes on 15 L NRB or 8 tidal volume breaths on 15L NRB will accomplish this.

With the difficult to ventilate patients you can use a PEEP Valve on the ambu bag. Respiratory therapists have access to the PEEP valves.  If you also put a nasal cannula on the patient (15 liters thru the cannula =passive apneic oxygenation) You in affect are giving CPAP.

BVM ventilate with low pressure(<25mm hg), low volume(6 ml/kg) and low rate (6-8/min).  It is also important to use a two handed thumbs down technique to hold mask on face.

Our ED has an awake look intubation kit in the omnicell.   It includes 4% lidocaine to nebulize and spray with mucosal atomizer, viscuous lidocaine also is included to put in back of throat.

NAP 4 Data: Higher mortality in ED and ICU, ETCO2 is the standard of care, awake intubation was not use when indicated, Failure to plan for failure, obesity was independent risk factor in a large %age of airway deaths.

Joan Coghlan made a great point that LMA is a great bridge device and a great device to help ventilate the difficult airway patient.

References for this talk:

1. Weingart SD, Levitan RM. Preoxygenation and Prevention of Desaturation During Emergency Airway Management. Annals of Emerg Med. 59, 3: 165-175
Second is the executive summary for NAP 4 and/or the full report (website)

 

WALCHUCK   RECTAL FB/PRIAPISM

You gotta get these out.  Never leave in place and wait for spontaneous passage

Look for signs of perforation on imaging studies.

Sedation may be helpful to remove.  Viscous lido can be used to help lubricate. Have patient in Sims or lithotomy position.  Sim’s position is with pt on their side with superior leg flexed at hip and knee.

Can attempt to place one or more foley balloons proximal to fb to remove fb.   .  Many patients will require GI to scope them to remove FB.  Sharp or  broken objects require surgery

Observe patient for 4-6 hours after removal to see if any signs of perforation develop.

 Priapism: corpora cavernosa become engorged with blood,  painful, can be due to sickle cell disease/thalassemia/leukemias, many other pharmacologic causes, erect penis with flaccid glans, tx with terbutaline in deltoid muscle, ice to perineum/penis/scrotum, narcotic analgesia, penile block at 11 and 1’oclock positions, aspirate the corpora at 10 and 2 o’clock, instill phenylepherine/saline, cardiac monitoring, for sickle cell patients exchange transfusion is indicated.

Penile fracture: rupture of the tunica albuginia, u/s is helpful for diagnosis, treatment is surgical, pt should not be sent home to follow up as an outpt.

WATTS      CENTRAL LINE PLACEMENT

TIP: Stretching the guidewire wil straighten the curved tip.

Avoid air embolism by flushing all lumens of CVC with saline prior to puncturing skin.  Also keep thumb over hub of needle when it is in the vein.

Avoid wire embolism but not using force to pull wire out.  If you meet resistance, remove needle and wire together as a unit.

WASH YOU HANDS PRIOR TO PROCEDURE!   USE STERILE TECHNIQUE INCLUDING GOWN/GLOVE/MASK/DRAPE.

Trandelenburg is important to distend the IJ.  For IJ central lines, turn the patient’s head to the left but over rotation can increase the risk of arterial puncture.  

NEJM video was shown demonstrating proper placement of IJ central line.

Femoral lines are the highest risk lines so try to avoid them.  CVP measurement is not reliable with femoral lines.  

Supraclavicular approach for subclavian vein: Puncture skin one finger breath lateral to SCM and superior to clavicle. Aim toward contralateral nipple.  

Harwood Comments: Look with U/S prior to draping patient and getting sterile.  The RIJ may have an old clot precluding that site.   Getting in the central vein requires a confident jab or poke through the vessel wall.     You don’t need to place the wire all the way into the vein to just pull it back to thread thru the catheter.  You can put the wire in part way and have the external wire to thread thru the catheter.

Coghlan: comments: Line up needle bevel  with  numbers on syringe so you know what direction your bevel is oriented when it is in the patient.  With the subclavian approach if you turn your bevel downward after getting in the vessel will direct the wire into the chest.

Place catheter about 15 cm(14-20cm) into the chest.   You should make an estimate prior to placing catheter.  Extremes of body habitus will affect this distance.

Never let go of the guidewire!

ERIKSON   GU EMERGENCIES

 Balanoposthitis: evaluate for diabetes, treat with retraction of foreskin and cleansing with soap and water, topical antifungals or oral fluconazole. Some cases may require anti-staph antibiotics.

Phimosis: foreskin cannot be retracted. Treat with hygiene and topical steroids.  If pt cannont void then emergent surgical procedure is indicated.

Paraphimosis: inability to reduce retracted foreskin.  Tx with manual reduction, multiple small needle punctures of glans to release edema fluid.  Can also try compression/ice cooling to decrease swelling.

Testicular torsion: cremasteric reflex is unlikely to be normal with torsion but this sign is not 100%.  Gotta get a testicular u/s when considering torsion in the diagnosis.  Treatment is surgery.  Manual detorsion may be indicated.  “open the book” is the way to think about how to reduce torsion.  This is most commonly successful motion. If pain worsens then stop and try to detorse in opposite direction.

Torsed appendix tesis: blue dot sign. Treamtment with NSAIDS

Epididymitis: gradual onset, fever, dysuria, urethral symptoms.   Pyuria in 50% of cases,get cultures of urine and urethra,  r/o torsion.   Tx with antibiotics.  If sexually active treat with rocephin and doxy.  If likely coliform source, give bactrim.

Fournier’s  Gangrene: Polymicrobrial infection, imunocompromised patients,  pt’s will have pain and look sick, check for crepitence in genital area, treat with big gun antibiotics Imipenem and Vanco. SURGERY is required.

Zipper entrapment injury: Cut bar of zipper

Harwood comment: Is Nair an option for hair tourniquet?  Many people felt this would be irritating to skin.  Also discussed was using sugar to decrease edema of a paraphimosis.

Conference Notes 5-15-2012

Conference Notes 5-15-2012

Patel/Collins    Oral Boards

Case #1:  Hemophiliac with an intracranial hemorrhage.   Give factor 8 to get level to 100%=50U/kg.  Give factor 8 prior to CT imaging.  If compartment syndrome suspected don’t check pressure  until after giving factor 8. 

Case #2: Splenic injury with intraperitoneal bleeding.   Diagnose with FAST exam.  Treat with fluid and PRBC resuscitation.   Spleen is the most common organ injured due to blunt abdominal trauma in both adults and kids.

Case #3:  Firefighter  exposed to heat and smoke.  He has airway injury, burns, and CO exposure.   Have to know the Parkland Formula for burns.(4ml/kg/%BSA burned; half given in first 8 hours post injury, 2nd half given over the next 16 hours)  Treat CO exposure with 100% FIO2 and get hyperbaric therapy arranged.   Treat with pain meds/update tetanus status.     CO is the most common tox cause of death.    Smokers can have a baseling CO level up to 10%.   Give hyperbaric therapy for syncope, confusion, seizure, neuro deficit, cardiac ischemia or level more than 25% in a normal adult or more than 15% in a pregnant patient.

GROMIS   M AND M

Flash Pulmonary Edema:

STuPID HPI: Surgeries/Trauma/Pain or paresthesias/Infection or fever/Drugs or toxins.  Ask this in a format to get yes or no answers.

Communication is vital during a resuscitation.  You have to take leadership of a code situation and designate team members to certain tasks.   Consider what the ramifications of your action will be prior to taking an action.

Dan discussed the multiple utilities of using ultrasound in the crashing patient.   Do RUQ view, suprapubic view, sub-xiphoid and parasternal long views.  

There are cardiac and non cardiac causes of Flash pulmonary edema.   Non-cardiac causes  include ASA or opiate overdose and HAPE.

Sgarbossa  Criteria for AMI in LBBB: 5mm of discordant ST elevation anteriorly,  or 1mm of concordant st elevation or depression in any lead.

Comments from Joan Coghlan:

First I would reiterate what all the attendings voiced:  this wasn't a case to feel chagrined about.  It was a sick lady on the cusp and she was going to get sicker no matter what was done.  In fact she survived because of your actions.
 
Just in terms of approach to the acutely dyspneic patient:
As true of every patient and every condition, stay diagnosis-oriented.
SOB?  Listen to lungs -- should get a good feel if it is COPD (quiet, no air movemnt), pneumothorax or effusion (decreased on one side only)  or CHF (rales) or noncardiogenic pulm edema or pneumonia.
 
If lungs seem clear and well aerated, then consider
PE
Angina
Tamponade
Arrhthmia
Valvular disease, aortic stenosis/regurg
Septic Emboli to the lungs
Generalized sepsis
Lymphangitic spread of undiagnosed cancer
Sarcoid, TB, etc.
 
Or  tox/metabolic like DKA or lactic acidosis or ASA causing compensatory resp alkalosis and fatigue.   Anemia  also may cause some low grade DOE, though not severe like this pt.
 
 
The point is stay DIAGNOSIS- ORIENTED  and use your physical exam and cxr, ABG to help you systematically rule each in or out.
 
Also remember if you decide the pt has copd or chf, you need to consider what CAUSED the pt to go into that state.  Don't just stop at that condition, find the DIAGNOSIS.
 
Once you established the pt is in Pulm edema, consider the causes of pulm edema --
 
1.Acute ischemia/MI    
 
   2. Arrhythmia -- this pt clearly had p waves on EKG but sometimesyou can miss slow VT when pt has those wide complex LBBB
 
3. Acute valvular incompetence due to ruptured papillary muscle/MI or to aortic dissection into aortic root (or endocarditis)
 
4.  Hypertensive emergency.   
5. High output failure from thyrotoxicosis or anemia (maybe beri beri or something like that)
 
6.  Acute myocarditis/ cardiomyopathy
 
 
Third, in addressing the fluid bolus, I totally agree with Christine and Elise, I would have given fluids.  I don't consider that as a mistake.  Again this lady decompensated due to her disease process not something you did.
 
BUT ask yourself what DIAGNOSIS you are treating if you give fluids --  the patient got hypotensive while you were in the process of discerning the cause of her acute dyspnea. Ask yourself why pt got hypotensive (dont just shoot from the hip as Gromis mentioned; ie an automatic reflex, hypotension = fluids) 
 
Lets get DAIGNOSIS-ORIENTED in deciding what to do for that hypotension:
 
1. sepsis  -- fluids
2. tamponade -- fluids
3. Pe with right heart failure -- fluids
4. pneumonia and dehydration -- fluids
5. vasodilation due to meds like ntg/morphine  --- fluids
6. tension pneumothorax --- NOT fluids ( REassess pt when RN says they got hypotensive, look at them, listen to lungs, check trachea for deviation, look at vent, make sure IV didnt infiltrate or central line got disconnected and pt is bleeding out --for real)
 
Many of the diagnosis you are contemplating are treated with fluid boluses.
And when you look at the diagnoses that may not especially benefit from fluid boluses, you are probably going to intubating them anyway because their disease state is going to follow its natural course which will need ventilatory support.     Once these patients are on the ventilator,  oxygenation is not a problem. So give them the fluids to expand their intravascular volume, fill their right heart because they may need that and if you get behind the 8-ball on that there is no coming back.  Conversely, if you overshoot on volume, you can intubate and support them through some diuresis.   ARDS is going to be the only major problem with oxygenation and again, somebody with ARDS has much greater problems.
 
So, my few comments became diatribe but I think we do best when we stay DIAGNOSIS-ORIENTED  and tailor and modify our treatments based on those assessments.   I would have done the same thing Gromis did and I would still do that today.  Don't fear the fluid. (fear the reaper).
 

WORKSHOP: CRITICAL CARE NURSING PROCEDURES

 

 

Conference Notes 5-8-2012

Conference Notes 5-8-2012

RYAN         MUSCULO-SKELETAL STUDY GUIDE

Medial meniscus injuries present with clicking, locking or pain with extension of knee.

Osteoarthritis: Gradual onset, pain is moderate.  Joint not clearly “hot”.

>50,000 WBC’s on joint aspiration points to septic joint.   This is a boards type cut off.  Real life is less clear cut but it is a guideline.  If you have a high suspicion of septic joint and the WBC count is less than 50,000 you should still culture the aspirate and consult ortho for close follow up or possible admission.

Sciatica can linger for 8 weeks duration.

Cauda Equina causes acute urinary retention and overflow incontinence.  Also look for saddle anesthesia and lower extremity weakness.

Spinal stenosis: Pain with walking due to neurogenic claudication.  Patient walks with anterior flexion of waist  to reduce traction on spinal cord. 

Empty can test specifically evaluates the supraspinatus muscle  of the rotator cuff.   The rotator cuff activates abduction and int/ext rotation of shoulder.

Foot puncture wound thru a gym shoe, prophylax  for pseudomonas.    If puncture wound thru sock or bare foot prophylax for staph/strep.  Get xray for foot puncture wounds.  Have high index of suspicion for fb especially if pt has fb sensation.

Treat a felon with a longitudinal incision at area with most fluctuance.

Carpal tunnel syndrome: Risks include obsesity, pregnancy, dm.   Phalen’s and Tinnel’s tests.  Tx with splint and analgesics.  Refer to ortho.

Amputated digit:  wrap in saline gauze, put in plastic bag, place bag on ice.

Finklestein test evaluates for De Quervan’s tenosynovitis.

 

CHASTAIN/BAROUNIS      STEMI CONFERENCE

Think circumflex lesion with minimal inferior ST elevation, tall R waves anteriorly and lateral st depression and mostly anterior st depression.   Circumflex lesion=posterior infarction.

Give 2b3a inhibitors in patients with chest pain and not STEMI but has ST depression or dynamic EKG changes.

Cardiac patients who are unstable get kicked out of STEMI bundle.  Resuscitate them first.

Consider strongly balloon pump in the patient with cardiogenic shock.   Before placing a balloon pump, you have to exclude aortic dissection.  

Neuro events can cause an adrenergic output that can make the cardiac apex ischemic.  You can see ST segment elevation with no reciprocal changes.

STRASBURGER-VILLANO    ORAL BOARDS

Case 1: Optic Neuritis due to MS.   Usually a monocular condition.  Painful with eye movement.  Treat with IV steroids (has to be IV not po steroids).  Be sure to get visual acuity on all eye related cases both for the boards and in real life.   

Case2: PEA cardiac arrest due to variceal bleeding and pneumonia.  

.Case3:  Dislocated patella.    Treat with passive extension of knee with firm pressure on patella redirecting it to the normal position.     There was debate about the need for moderate sedation.  Some felt pain control with no or light sedation was adequate.

CARLSON    TOXICOLOGY    MEDICATIONS FOR DIABETES (Sorry I missed the beginning portion of the lecture)

IV d5 or d10 drips give miniscule amounts of glucose.  To replace glucose more robustly, feed pt if at all possible.   If you need IV dextrose you will need to give 1 or more amps.  

Octretide can be used for sulfonylurea and meglitinide toxicity   

Admit: Any long acting insulin OD, intentional insulin OD, recurrent hypoglycemia, sulfonylurea/meglitinide, hypoglycemia related to significant change in renal function or liver function.

Glucophage can cause MALA (metformin associated lactic acidosis).   It interferes with normal cellular aerobic  metabolism.   Mechanistically looks like a mild/non-fatal  cyanide overdose.   Can occur in mono-overdose.    Treat with hemodialysis.   Fatal cases prognostic factors are low ph 6.9 or lower, lactate over 25 or metformin level over 50.    Should dialyze if ph around  7 or heading downward.

Avandia/Actos: increase insulin sensitivity and decreases glucose production in liver.   Doesn’t cause hypoglycemia in OD.

Januvia: Stimulate insulin release with an elevated glucose.   Has not been shown to cause hypoglycemia in OD.

Byetta: Glucagon-like peptides.   Stimulate glucose dependent insulin release in gut.   Does not cause hypoglycemia in OD.

Victoza: similar mechanism to Byetta.  Can cause pancreatitis.

Symlin: Amylin agonists.   Slows gastric emptying, decreases gluconeogenesis, increases satiety.  No reports yet of hypoglycemia.

Insulin/Sulfonylureas/Meglitinides/Biguanides:  These all can cause hypoglycemia in overdose.  

MISTRY        ROOM COVERAGE PROPOSAL

PUTMAN          THERMAL BURNS IN THE ED

Stop the burning process by removing any object such as clothing or rings/watches that can retain heat and further burn or produce a tourniquet effect.

IV fluid resuscitation with Lactated Ringers

Treat with appropriate pain medications.

Get further history:  chemicals/closed space/explosion/CO/cyanide/electrical injury.

Burn-specific secondary survey: eval for inhalational injury.   Intubate for early signs of airway injury.

Estimate body surface area involved in burn with rule of 9’s,  palm,  Lund-Browder chart.

Burn depth: first=erythema of epidermis only,   2nd=blistering, 3rd=thru epidermis and dermis involving nerve endings and should have no pain in area of 3rd degree burn. 4th= involve deeper structures such as tendon and bone.  

Intact blisters can be left alone.

Transfer criteria:  probably need a check list to remember all of them.

Escharotomy for circumferential burns that are causing ischemia.

Skin is burned by temperatures over 113F.

Cellular Na pump is disrupted by burns.   Depression of cardiac contractility can be caused by burns.  Lactic acidosis can occur from burns.

Fluid resuscitation:  Parkland formula is  4ml XKG X %BSA burned= volume.   50% given in first 8 hours after injury (no ED arrival).   Remaining 50% given over the following 16 hours.  This is a guideline that can be altered based on urine output/cvp/pulmonary status.  Peds patients also need weight-based maintenance fluids.

Half of all fire related deaths are due to smoke inhalation.

Think cyanide for fires in which wool, silk, polyurethane have  burned.

IV antibiotics not indicated for prophylaxis of burns.   Only use if infection is evident.

Tegaderm or duoderm can be used as burn dressings for smaller burns.

Pregnant burn patients should have fetal monitoring for viable age gestations.

If the Burn Center asks for a photo of a potential transfer patient, do not identify the patient with any facial views.    Document in the chart that patient consented to the photo.

Conference notes 5-1-2012

Conference Notes 5-1-2012

SCHROEDER/CHANDRA   ORAL BOARDS

Case 1  Eczema herpeticum.  Treat with anti-virals and anti-staph antibiotics.   Acyclovir takes mortality down from @10% to 0%.  Can complicate eczema.   Staph is a frequent co-infection.   Eczema herpeticum is more likely to be painful lesions in contrast with impetigo. Diagnosis more likely in patients taking immunomodulators for eczema.    DO NOT GIVE STEROIDS!   

 

Case 2   Fish hook embedded  in thumb.  EP should remove fishhook, examine for nerve or tendon injury.  Irrigate wound if possible. Consider prophylactic antibiotics.  Check tetanus status.

 

Case 3 Cervical Epidural Abscess.  MRI is superior to CT for diagnosis.  Consider in pt’s with: iv drugs, dm, steroids, invasive procedures, trauma, immucompromised.   Consult neurosurgery.  IV antibiotics.

SMALL GROUP SESSIONS PEDIATRIC MEGACODES

  1. Peds SVT : treat with adenosine 0.1-0.2mg/kg, next amiodarone 5mg/kg.  If unstable can cardiovert 0.5-1j/kg.
  2. V-fib: Defibrillate with 2j/kg double dose if unsuccessful.  Epi  0.01mg/kg, amio 5mg/kg.  Post resuscitation avoid hyperoxia and consider cooling.
  3. V-tach: synchronized cardioversion 0.5-1j/kg,  if failure then increase to 2j/kg.
  4. Hypoglycemia and shock: 0.5-1g/kg dextrose which means D25W: 2-4 mL/kg, D10W: 5-10mL/kg.  For hypovolemic shock give repeated 20cc/kg boluses

 

LAM  PEDIATRIC U/S APPLICATIONS

Use a high frequency 5-10MHz probe

Graded Compression: slow gently increased compression on abdomen.

Appendicitis:  Appendix is medial to psoas and anterior to iliac vein.  Target sign on transverse view.  Non compressible, Fluid collection, Target sign, Diameter>6mm (mnemonic is NFTD nothing further to do)

Intussusception: Use graded compression.  Follow the expected contour of the colon.  Again look for a target sign.

Pyloric Stenosis: find stomach first and go to pyloris.  Anterior and lateral to aorta. Abnormal pyloris is too thick or too long.

BAROUNIS  ABG BASICS

The 4 step approach to acid base disorders

 

Step 1: Get the labs (VBG=ABG), you need Na, Cl, HCO3, pH and PCO2

 

Step 2: Calculate the anion gap (Na - (HCO3+ Cl) Normal is < 15, abnormal > 15

 

Step 3: RULE of 15, the PCO2 and the last two digits of the pH should be the bicarb + 15. 

ie- if bicarb is 15, PCO2 should be 30 and the pH should be 7.30

3 Possibilities of PCO2:

1. it is what it should be (simple wide gap metabolic acidosis with respiratory compensation)

2. The PCO2 is lower than it should be (patient is breathing faster) primary respiratory alkalosis

3. The PCO2 is higher than it should be (patient is breathing slower) primary respiratory acidosis

 

Step 4: 1:1; Normal bicarb - 24, normal gap = 15

The CHANGE or increase in anion gap from baseline should = the change or decrease in the bicarbonate

ie if the anion gap is 30, the change or delta gap is 15 (30-15=15) therefore the bicarb should decrease by 15 (24-15= 9)

3 possibilities of bicarb:

1. It is what is should be (simple wide anion gap acidosis)

2. The bicarb is LOWER than it should be (in the above case if the bicarb was 5 instead of 9) additional primary non-gap metabolic acidosis

3. The bicarb is HIGHER than it should be (in the above case if the bicarb was 15 instead of 9) additional primary metabolic alkalosis

 

 

Remember at SEVERLY low bicarbs the pH and the pCO2 will be less reliable. the PCO2 is not +15 when bicarb <5, it is 15. 

 

Sorry I went a little fast and I can send out the ppt later if you want to practice the cases.

 

VILLANO   ABCD’S OF HYPOTENSION AND BRADYCARDIA IN TOXICOLOGY

Alpha Agonists, Beta Blockers, Calcium Channel Blockers, Digoxin

Alpha Agonists (clonidine): centrally acting antihypertensive.  Onset  30-60 minutes after ingestion.  Pt will get hypotensive.   They wil have respiratory depression, lethargy and coma, miosis.   Treatment is supportive.   Try narcan, it may help.   This overdose will look somewhat like an opioid overdose with more hypotension.

Beta blockers:  Usually symptomatic by 2 hours.  Look for hypotension, bradycardia, and early altered mental status.   Seizures are possible.     Pt’s can have normo to hypoglycemia and mild hyperkalemia.  Tx with atropine,  glucagon (activates g protein that increases cyclic amp by alternate no beta receptor pathway) 5 mg over 5 minutes.   Glucagon may induce vomiting.  Tx also can include calcium, pressors,  hyperinsulinemia-euglycemia ,  intra-lipid can be tried in crashing pt.   Pacing and balloon pump may be required.   Can d/c to psych for immediate release form and asymptomatic after 6 hours.  All other admit to tele or icu if abnormal vitals.

Calcium Channel Blockers: Dihydropyridines cause decrease in smooth muscle tone andlower bp.  Monohydropyridines affect cardiac conduction and cause bradycardia.   Look for hypotension, bradycardia, hyperglycemia, late mental status changes, acidosis.  Treat with Calcium  (1g chloride central, 3g gluconate peripherally) , iv fluids, iv atropine,  insulin-glucose therapy (insulin is a pressor/response may take 60 minutes/bolus 1u/kg and infuse 0.5-1u/kg/hr/titrate to bp>90/give D50/hypoglycemia doesn’t happen as much as you would think), glucagon, intra-lipid.  Have a low threshold to put pt in ICU.

 

Digoxin:  Recently covered in a previous conference.   Look for nausea/vomiting and arrhythmia.

 

 

 

Conference Notes 4-26-2012

Conference Notes 4-26-2012   ICEP Spring Symposium

Research Presentations

Dr. Phillips: Absorbable sutures are a lower cost method compared to non-absorbable sutures to repair wounds.   No difference in scar outcome, and similar complication rates.   Also easier on the patient.  No need for suture removal.

Dr. Cambride: Intubation during CPR (manual vs. automated).  Success rate and time to intubation was no different when compressions were or weren’t being done.  No difference between manual or automated compressions either.   So don’t stop compressions to intubate.

Dr. Hartman: Patient follow up requirements are carried out in widely different manners across the country.  Program Directors are generally dissatisfied with patient follow up learning methods.

Dr. Rifenburg: PE patients presenting with syncope are more likely to have a saddle embolus and less likely to have a small embolus than PE patients without syncope.  PE patients with syncope were more likely to have EKG changes.  PE patients with syncope were more likely to have RVH than non-syncope patients.

DR. CANTOR   MISTAKES YOU DON’T WANT TO MAKE-PEDIATRICS

Investigation priorities in seizures: infection, mass, metabolic (electrolytes), toxins.   Dilutional hyponatremia can occur from not mixing formula correctly.  Genital ambiguity suggests congenital adrenal hyperplasia.  

Progressive causes of altered mental status: mass, meningitis, opioids, hypoglycemia.  All must be treated emergently.    Don’t give full  dose narcan to heroin addicts.   Beware of hypoglycemia.  It can mimic any neuro deficit.  If you can’t talk to someone, check their blood glucose. 

Anticholinergics      vs.  Sympathomimetics= Dry skin    vs.  diaphoresis

Pheochromocytoma can mimic sympathomimetic overdose.

Psoas abscess can cause limp.   If work up of febrile limping patient with leukocytosis is not fruitful, consider ct abd/pelvis to look for abscess.

Encephalitic patients should be treated with acyclovir.  Look for cold sores to suggest HSV encephalitis.   Pt’s can have seizures with HSV that appear like unusual behavior or agitation. 

Common cyanotic heart lestions: truncus, transposition, tricuspid atresia, tetrology of fallot, total anamolous return.   Tetrology is the one that can present with a patient 1-2 years old.    Failed hyperoxia test strongly favors cyanotic heart disease

Dr. Thompson      Toxicology: What’s New in the Street

 Sorry I missed this one, talking with old friends.

Dr. Allen   To DNR or CPR

1960 was the first description of modern CPR (ventilation,chest compressions, defibrillation).   1974 were first ACLS Guidelines.

22% overall rate of survival from CPR in ED’s across the country.  This is better than any other setting.  In general, survival is around 17% in all settings.

EP’s have reservations about CPR: overhyped, can cause suffering, costly, benefit is mostly extra days alive but not quality of life.

Slow CODE is not acceptable.    It is deception and could lead to survival with poor neurologic outcome.

APPLE   Shared Decision Making with Patient/Family at End of Life: Awareness , Prognosticate (make an estimate of short term outcome), Plan (basically figure out the patient’s goal and it is ok to give recommendation),  Lay the ground work, Empathize (sit down, be quiet and spend time listening, express regret, hopeful attitude)

NEXT Lectures

Dr. Kegg: Within 6 hours CT brain may have a high enough sensitivity to rule out SAH.

Dr. Mehan: Itralipid for local anesthetic toxicity acts as a lipid sink, metabolic substrate, or direct activation of CA channels.   Can also be used for beta blocker and CCB, TCA, Atypical antipsychotics and buproprion  Od’s.  Downsides: ARDS, pancreatitis, has compatibility issues with other meds.

Dr. Oh: You can use IVC/Ao ratio to evaluate dehydration in pediatric patients with diarrhea.  Serum bicarb over 15 is reasonable cut off for non-serious dehydration.  ETCO2>34 ruled out HCO3<15.  Check glucose because hypoglycemia is present about 10% of the time in pediatric patients with diarrhea.  Can give up to 60ml/kg bolus of saline for more severe dehydration.   

Dr. Pirotte: CDC and NIH say we should not due femoral lines (cat 1a recommendation) unless we have no other options.   They are more likely to cause infection.  Complications are worse, retroperitoneal hemorrhage, DVT, fistula, pseudoaneurysms.   Consider IO line in place of doing femoral line.

Dr. Rushforth:  Tranxemic Acid (TXA)will be more likely to be used in Trauma patients due to efficacy reports, no evidence of theoretical complications, inexpensive.   Most studies on this drug were done in under-developed countries where the baseline mortality may be higher to start with and critical care resources are less than in the US.  

Dr. Vogt: Scombroid most common around Florida and Hawaii.    Supportive care is the indicated treatment for all common fish toxins.  Ciguatera symptoms can persist for weeks to months.  Mannitol is still controversial as a treatment for ciguatera.  Alcohol can cause recurrence of ciguatera symptoms.  Be aware that fish flown in from other areas can cause various fish toxidromes.

Dr. Williamson:  Allergy to shellfish is due to shell proteins and not iodine .  Patients with shellfish allergy can receive radiology contrast.  Scombroid poisioning is due to bacterial overgrowth of improperly stored fish producing histidine.  The histidine is converted to histamine.  Pt’s will have flushing, rash, palpitations.   Treat with antihistamines.   Fugu is due to tetradotoxin that binds to sodium channels. Treat with supportive care and charcoal. 

Conference Notes 4-17-2012

Conference Notes 4-17-2012

PAQUETTE   STUDY GUIDE SPECIAL PATIENTS

One year risk of death after a fall in the elderly is 50%.

Heroin users: Cotton fever is due to unknown pyrogen effect.  Thought to be due to particulate matter from cotton or gram neg endotoxin.  However,  even patients who don’t use cotton to filter their heroin get cotton fever. So cause is unknown.  Fever resolves spontaneously.

Patients over 65yo account for 30% of trauma deaths even though they only account for 20% of total trauma patients.

Patients over age 80 have chest pain as the presenting complaint for AMI less than 50% of the time. SOB is more common.

Myositis ossificans (heterotpic ossification) usually found in chronically immobilized patients. Can be painful.   Usually around the hip/knee/femur/shoulder.

Post traumatic syringomyelia: ascending spinal cord lesion in a patients who’s neuro deficit had been previously stable.

STRASBURGER   HIV EMERGENCIES

1.2 million people in US are living with HIV infection.   1/5 people are unaware of their HIV infection.

17,000 people died from AIDS in 2009.

HIV deaths peaked around 1995.  

AIDS:  CD4 count less than 200 or an AIDs defining illness.  

CD4 count >500 is an immunocompetent patient.    CD4 200-499 is indeterminate so look for AIDS defining illness and consider CD4 %.

CD4% is a more reliable measurement than the CD4 count because it varies less.  Normal CD4% is 32-68%.

www.aidsmeds.com   is a good reference website for side effects and drug interactions of ART (anti-retroviral therapy).   UCSF post exposure prophylaxis hotline can help you make decisions for starting prophylactic therapy after body fluid exposure.   Other good website hivinsite.ucsf.edu.

General approach to HIV/AIDS patient: Find out CD4 count, CD4 trend, viral load, opportunistic infections/medication side effects.

AIDs patients who need CT: CD4<200, fever and  nuero findings or stiff neck.   Follow CT with LP.    If patient has a CD4 count >200 you can do LP without prior CT if no focal deficits or headache.    Absolute lymphocyte count= WBC count X %lymphocytes=>2000 then CD4 count is most likely over 200.  If less than 2000 CD4 count may be less than 200.  Have low threshold for doing LP in pt’s with neuro complaints.

Toxoplasmosis may have ring enhancing lesions on head CT.

Cryptococcus Neoformans: usually CD4 count is <100 but not always.  Have chronic meningitis picture or prolonged headache.   CT is usually unremarkable.  Serum cryptococcal antigen is 95% sensitive and may be useful in the patient refusing LP.  CSF cryptococcal antigen is 100% sensitive and specific. India ink staining is only 60-80% sensitive.   Treat with amphoB and flucytosine.  

Respiratory infections: most common are bronchitis or uri.   Keep guard up for bacterial pneumonia and PCP pneumonia.   Bacterial pneumonia due to strep pneumo is most common.   PCP is the most common opportunistic infection. 

PCP pneumonia: chronic cough, hypoxia with exertion.   Walk them on a pulse ox and see if their sats drop.   ABG can help if paO2 is <70 or AA gradient is >35 signifies severe PCP.   Serum LDH can help identify PCP pneumonia.   1/3 of patients will have a nl CXR.   Can also have a focal infiltrate. CT that does not show ground glass appearance is unlikely to be PCP.

Diarrheal illness: Get stool sample for lab testing.   CD4>200 and labs look ok, and they look well, they can go home on cipro/flagyl.   If not then admit.

CMV retinitis: CD4 count usually less than 50.  Blindness in 30%.  Fundoscopy shows cotton wool like findings.  ART does not reverse disease but just slows it down.

Retinal  varicella zoster can also occur in patients with very low CD4 counts

HIV patients have accelerated progression of  coronary heart disease.  HIV should be considered a CAD risk factor.     

Immune Reconstitution Inflammatory Syndrome (IRIS): Rapid CD4 cell count increase  or rapid decrease in viral load.  Can happen within several weeks of starting ART.   Due to underlying infection that the immune system was previously not responding to.   Treat infection like PCP prior to starting ART.  Steroids also can diminish the symptoms of IRIS.

Health Department Clinics can provide free HIV testing for patients.

ANGELA ROMERO    PERFECT SERVE

GARRETT-HAUSER     ETHICS

Elder abuse: 4 felony convictions in and Kane and Cook counties in past 18 months for physical abuse and neglect. 

NH Abuse: Psychiatric patients and and sexual felons in NH population.    2 elderly alzheimers got into fight and one died.  The NH owner and staff are being prosecuted for murder/manslaughter.  NH cannot give psychiatric meds (geodon) for dementia.

Case #1: Is withdrawing care the same as murder?   The right of a competent individual to refuse any form of care is absolute.  Power of attorney also has the right to refuse care for a patient.  In some states, surrogates have similar power to refuse care.   We have a withdrawl of care form to use in ED if you want to withdraw care.  Need terminal condition, permanent unconsciousness, incurable or irreversible condition.    So answer is no, withdrawing care is not the same as murder.  Withdrawing care is not the cause of death but rather returning patient to condition of illness that will result in death.    In a DNR situation, there is no difference in not initiating treatment and withdrawing treatment. 

Case #2  13 yo girl with vaginal bleeding and dangling umbilical cord with no fetus.   Pt will not say where  baby is.   Baby was in garbage dumpster.   To whom do you have a duty as an ER doc?  You have duty to patient and to baby both ethically and legally.  Long discussion on this case but consensus felt  probably smart move would be to place mom in psychiatric care rather than sending her directly to jail. 

Case#3    How do we manage patients with chronic pain issues and drug seeking behavior?  10% of general population has substance abuse problems.   Top 3 medical specialties  with substance abuse are EM, anesthesia, and psychiatry.  Oak Forest  Hospital has a free urgent care 24/7.  This clinic can give patients access to Cook County Clinics (pain clinic).

WILLIAMSON   NEXT LECTURE  FISH TOXINS

Anaphylactic Shellfish Reaction:  Type 1 IGE immune mediated response.  Can patients with shellfish allergy get IV contrast?   Yes very safe.   Shellfish allergy is to the shell not to iodine.

Shellfish poisoning:  4 syndromes diarrheal/ amnestic reaction/neurotoxic/paralytic (sodium channel blockade) multiple cases identified in Alaska.  Pt’s have to be intubated sometimes.

Scromboid: Bacterial overgrowth in improperly stored fish producing histidine.  Can be indistinguishable from allergic reaction.  Tx with antihistamines.

Ciguatera: activation of sodium channels.   Onset 3-6 hours after ingestion.  Symptoms are paresthesias, dental pain and paradoxical temperature reversal.   Iv mannitol can be tried.

Fugu:  Tetradotoxin binds sodium channels resulting in ascending paralysis.  Supportive care and charcoal decontamination of gut are treatments.

Stingray: Venom has no specific antidote.  Get an xray for retained fb. Give prophylactic levoquin.

Mercury: Fish consumption is most common cause.  Symptoms include fatigue,  sensory impairment.  Dimercaprol is treatment.

PATEL SAEM CPC CASE PRESENTATION

Neuromuscular causes of weakness: Brain, Cord, NM junction, Muscle disorders.

Periodic Paralysis causes acute weakness due to  channelopathy.

Familial Hypokalemic Periodic Paralyssis: white race,

Thyrotoxic Periodic Paralysis:  Acquired disorder, age 20-50, Asian males.  Carbo load can precipitate an acute episode.  Ekg may show U waves.  Treat with beta blockers and possibly potassium.   Hypokalemia and labs c/w hyperthyroidism confirms diagnosis.   

Conference Notes 4-10-2012

Conference Notes 4-10-2012   (Sorry for missing 4-3-2012 but I was in Atlanta for CORD)

BRAD KUTKA      CPC Case Presentation    

BEAU WILLISON/DENNIS RYAN   ORAL BOARDS

Case 1: Ascending Cholangitis :   Patient had Charcot’s Triad of Fever/Pain/Jaundice.   Reynold’s Pentad adds altered mental status and hypotension.  Treated with antibiotics and ERCP.

Case 2:Fitz Hugh Curtis.    Pt had PID Symptoms with RUQ pain.  Treated with iv Rocephin and Iv Doxy/ Azithromycin.

Case 3: Disseminated Zoster in a lymphoma patient on chemotherapy.   Treated with IV Acyclovir.  Panculture for neutropenia.   Cover with Cefipime as well.

ANDREA CARLSON    TOXIC MELTDOWNS

The hypothalamus triggers sweating in response to heat.  As you sweat, your serum osmolarity goes up.  At a certain point, high osmolarity will trigger the hypothalamus to decrease sweat losses to conserve fluids in body.

TCA’s can cause seizures, sweat gland dysfunction both factors in hyperthermia

Cocaine associated deaths are more frequent on hot days.

Hypothermia when sustained and severe causes denaturation of proteins and enzymes.

Hypothermia is the most important negative prognostic indicator for a drug overdose.

Serotonin Syndrome: Serotonergic medication AND  4 major or 3 major and 2 minor criteria.  Altered consciousness, mood change, coma, tremor, shivering, hyperreflexia, fever, rigidity, sweating, restlessness.  3 aspects of the syndrome: 1. Cognitive /behavioral  2. Autonomic changes  3. Neuromuscular findings.  Most common findings for these three arms are: confusion, fever, myoclonus. Common drugs: SSRI’s, amphetamines, Cocaine, tramadol, DXM, lithium, MAOI’s, methylene blue (multiple case reports), linezolid, metoclopramide, prochlorperazine.

Cyproheptadine is an old antihistamine.  Direct serotonin antagonist. Anecdotal evidence of improved survival in serotonin syndrome.  PO forms only available.

Neuroleptic Malignant Syndrome: due to dopamine deficit.  Slower in onset than Serotonin syndrome.  Altered mental status and rigidity precede hyperthermia and autonomic instability.   Bradykinesia and “lead pipe rigidity”  are key neuro findings.

For both SS and NMS you gotta cool the patient.   Control seizures,  give benzos, give Dantrolene (dantrolene is a skeletal muscle relaxant and can make a pt weak but not paralyzed).  Bromocriptine for NMS, Cyproheptadine for SS.   Use other cooling measures like fans and misters.  No reports yet of using cool guard to bring down pt’s temp.   

TONY GRIPPO   TRAUMATIC BRAIN DEATH

Uniform Death Act  1981 makes brain death equivalent to cardiovascular death.

Prerequisites: Know cause of coma, normothermia, no drug intoxication/poisoning.  ETOH < 80.  No treatable metabolic causes.   BP>100systolic.

Exam: Coma; completely unresponsive to noxious stimuli.     Absent Brain Reflexes; no papillary light reflex/no corneal reflex/no ocular movement to cold calorics or doll’s eyes/response to painful stimuli above the neck/no jaw jerk/no gag or cough with suctioning.      Apnea;Vent disconnected, o2 given via tracheal canula.  If after  10 minutes pCo2 of 60 or greater and no breath that is positive for brain death.  Need two tests to be definitive.   If apnea test is inconclusive you can do nuclear medicine testing or cerebral angiography.   Nuclear medicine scan shows “empty light bulb” and “hot nose” findings when positive for brain death.

Lazarus reflex: There are some spinal reflexes even though the patient is brain dead.  Neck flexion can cause upper extremity flexion even though brain death is present.   Cremasteric reflex is preserved. There are other toe and finger reflexes that persist after brain death as well.

You have to wait 72 hours after therapeutic hypothermia (some say 6 days) before you can start considering for brain death.

Brain dead patients are the source of 50% of renal transplants and majority of extra renal organ transplants.   

Supporting the brain dead patient to maintain organ perfusion  should include volume infusion, pressor management,  and  hormone replacement therapy.   Dopamine first  and  then vasopression if needed are  the recommended pressors.  Amiodarone is the recommended antiarrythmic.   Hormonal resuscitation includes T3, Vasopressin, Methylprenisolone, and Insulin.

MARK HINTON   M&M

Pt on Coumadin falls and hits head. Subtle mental status changes.  Initial CT was neg.  INR was prolonged to 5 range.  Pt came back 1-2 days later for increased mental status changes and ataxia and renal insufficiency.  Family notes on second visit that pt has  been having multiple falls.  Pt found to be digoxin toxic. 

Digoxin toxicity:

Sources of error: minimizing problems by patient.   Recall biases,  Attending/Resident discrepencies.  Anchor bias,  premature closure.

William Withering discovered digoxin in 1785.  Treated dropsy.

Foxglove, oleander, and lily of the valley are botanical sources of digoxin.  You can also get digoxin toxic from licking the skin of toads (also hallucigenic).

Digoxin is mostly eliminated renally.

Ekg changes with therapeutic digoxin level, lateral/inferior  st depression with downsloping st segment and inverted t wave.   

First symptoms of overdose are somnolence, dizziness, and confusion.

Causes of toxicity: overdose, renal insufficiency, and drug interations (amiodarone, verapamil, vytorin, other statins, macrolides).

EKG changes in toxicity: pvc’s, any svt with av block, pat with block is pathognomonic, bidirectional V-tach, sinus bradycardia, slow afib,  regularized afib,

Severity of hyperkalemia correlates with mortality.

Xanthopsia: yellow vision.   Did Van Gogh suffer from digoxin toxicity?  He painted his doctor sitting with a foxglove plant nearby.

Treat hyperkalemia with  standard treatement insulin/dextrose, bicarb, kayexelate.  Calcium now thought to be ok for hyperkalemia.    Digibind treats the digoxin overdose and the hyperkalemia.   Digibind works in about 30-60 minutes.   Digibind is safe in kids down to 12 weeks old.

 

Conference Notes 3-27-2012

CONFERENCE NOTES 3-27-2012

RESIDENT TOWN HALL MEETING

JIM MALETICH  5 SLIDE FOLLOW UP

Pt presented with antero-lat STEMI and stroke symptoms.   DDX:dissection/endocarditis/prior mi with clot and embolism.

CTA was neg for dissection.  CT head showed some sign of stroke.

TPA was given

1% of MI patients will have stroke in 24 days.   Anterior MI is a risk factor for stroke.

0.1% of STEMI’s are caused by dissection.

ADAM WISE   5 SLIDE FOLLOW UP

14mo child with fever to 39.7 dx’d with viral illness and dc’d.  Came back 5 hours later with purpuric rash and LP showed meningitis.

<3mo: ecoli, GBS, listeria.  Tx: Cefotaxime and  ampicllin

>3mo: strep pneumo, neisseria, staph aureus.  Tx: ceftriaxone and vanco

Relationship with parents and good instructions are key to making sure parents brought child back to ER.   Parents impression of child’s condition is more sensitive than your clinical eval sometimes.

PAARUL CHANDRA  WILDERNESS EM

Signaling for help: highest ground is best,  SOS three short/three long/three short, three fires in triangle shape, mirror signaling.

SAM splint is a versatile and mobile splint material.

When splinting in wilderness, try to gently realign fracture fragments.

Be sure to pad your splints because using ski poles or sticks is not comfortable for patient.

SPLINTING LAB WITH MAKE SHIFT MATERIALS

ANNA LOUGHLIN   DISLOCATIONS

96% of shoulder dislocations are anterior

Hill Sachs Deformity: impaction type fx on humeral head

If pt has an associated humeral neck fx with dislocation there is increased risk of avascular necrosis.

Intra-articular injection of 20ml of lidocaine can provide adequate anesthesia to perform reduction.

Posterior shoulder dislocation associated with seizure and lightning strike.  Much less common. Subtle signs of dislocation. Light bulb sign is when humeral head has the contour of a light bulb.

Luxatio Erecta: Inferior shoulder dislocation. Humerus is directed upward. Reduction is traction/counter traction

90% of hip dislocations are posterior.   Look for other injuries because these are high energy dislocations.   Patients can have sciatic nerve injury with posterior dislocations.

Femoral nerve injury can occur with Anterior dislocations

Shenton’s line: continuous curve from femoral neck to superior pubic ramus.  If not continuous suggests dislocation.

Hip dislocations do better with very prompt reduction.  It is an emergency.

Knee dislocation is a true ortho emergency as well.  Got to look for popliteal artery injury and check ABI’s.  ABI’s should be >0.9.  If ABI less than 0.9 then get angio or CT angio.   Tibial nerve or common peroneal nerves can also be injured.

JIM MCKEAN  OSTEOFIBROMA

When kids have chronic dull knee pain look for malignant lesions with xray

BRIAN KESSEN

Pt coded.  After resuscitation EKG shows IRBBB.  PE considered and pt sent to CT.  Pt had recent SAH.

Chest CT showed bilat pe.  Ct head shows improved SAH.  Heparin started, pt was 13days after SAH. 

Pt  later had IR thrombectomy.  She did well.

EKG findings of PE: S1Q3T3,  Anterior T wave inversions,  IRBBB and tachycardia.    EKG has relatively poor  sensitivity/specificity for PE.

TPA contraindications mnemonic: TPA IS BAD

Trauma, pericarditis, active bleeding, intracranial pathology, bp >180, allergy, dissection

CHRISTINE KULSTAD  JEOPARDY  TRAUMA

Lactate is a good marker of resuscitation status.  Base deficit can be used similarly with increasing negativity correlating with worsening outcomes.

Perimortem c-section should be done within 5 minutes of cardiac arrest.  Use large midline incision.  Make small uterine incison and extend with scissors

Pregnant woman with pneumothorax put tube in a higher rib space than normal because diaphragm is higher.

 Chest tube output: 15ml/kg  immediate   or 4ml/kg/hr  indicate need for surgery in kids.   In adults it is 1500ml immediate and 200ml/hr/4hours.

1:1:1 units prbc’s/ffp/cry is indicated for massive transfusion.

Antibiotics are indicated for open depressed, sinus related skull fractures and pneumocephalus. Tx with rocepin and vanco.

Head injury in pt on Coumadin.  Should always check INR. If INR is supratherapuetic, consider observation.

Impending herniation: Tx with mild hyperventilation with pco2=30-35, mannitol 0.25-1g/kg

Retrobulbar hematoma:  Retina can only tolerate ischemia for 90 minutes.  Do lateral canthotomy.

Zygomaticomaxillary fx: check facial sensation in multiple places, good eye exam, check jaw movement, extra ocular movement.

Fx of superior roof requires admission because it risks brain injury.   Fx’s of inferior and medial orbital walls can be repaired up to 2 weeks after injury.

Open jaw fx’s should get IV antibiotics and Oral Surgery consult, and admission.

Anogenital reflexes:anal wink, cremasteric reflex, bulbocavernosis reflex.

Neurogenic shock is hypotension due to cord injury,   Spinal Shock is physiologic loss of reflexes below spinal cord injury

Central cord syndrome: upper extremity more weak than lower extremity,

Brown Sequard: ipsilateral paralysis  and loss of proprioception,  contralateral pain and temperature loss

Flexion tear drop fx is unstable and requires surgery

NEXUS Criteria:  Midline posterior tenderness, intoxication, nl mental status, focal neuro deficits, intoxication

Canadian Rules: Severe mechanism of injury, age>65, extremity paresthesias,  able to rotate head 45 degrees.

Strangulation injuries should probably be admitted to observe for noncardiogenic pulmonary edema and delayed blunt laryngeal  (pt will have hoarseness and significant anterior neck pain) injury.

Carotid injuries can cause Horner’s syndrome.  

BRIAN FORT  DENTAL EMERGENCIES

32 teeth in the adult mouth.   Start counting at upper right posterior molar.   Finish at lower right posterior molar.  

Primary teeth 6 months to 6 years.

Pericoronitis: inflammation of gingival overlying erupting tooth.

ABX for mouth organisms use PCN, Augmentin, Clindamycin.

Dental Caries and Pulpitis: Tx with abx and pain meds

Temrex in our dental box can be used to cover dental carie.  

Apical Periodontitis: fluid or abscess around root of tooth.  ABx treatment

Ludwig’s Angina: infection around teeth can extend into soft tissue planes in neck.

Periodontal abscess: starts from gingival margin.   ABx, drainage, pain meds

ANUG: due to anaerobes.  Patients with lowered immune response are more prone to this.   Also stress, poor sleep and other things can predispose.   Pain/ulcerated interdental papillae/bleeding gingiva.  Cholorhexidine rinses and abx.

Postextraction Pain:  less than 48 hours post op is self limited. Treat pain.  Greater than 48 hours think dry socket.   Topical anesthesia, irrigate socket,  oil of cloves/eugenol in paste or ribbon gauze gives immediate relief.

Post extraction bleeding: Tx with gelfoam, surgical, hemecon.   Local injection of lido with epi.

Ellis classification of dental trauma not used by dentists.  But ER boards do so, Class 1 enamil, Class2 dentin, Class 3 pulp.

Cover ellis 2 and 3 fx’s with calcium hydroxide paste.

Splint luxated teeth or root fx’s with Coe-Pak (zinc oxide)

Intrusive luxations should be left alone.  Don’t pull the tooth out further.  The dentist will wait to see if tooth moves down over several days.

Reimplant avulsed teeth as soon as possible.   Fluids to preserve tooth: hanks>saliva>milk>sterile saline.

Avulsed primary teeth, leave them out.  

 

 

Conference Notes 3-20-2012

CONFERENCE NOTES 3-20-2012

LAST WEEK WAS ORAL BOARDS

CARLSON   ORAL BOARD DEBRIEF

Ductal Dependent Lesions can present with cyanosis or shock.   Mottled skin is prominent in these infants.   You have to consider sepsis and do full septic work up/give abx.  Lack of fever does not exclude sepsis in infants.  Prostaglandins are indicated.  Prostaglandins can mimic sepsis.    Prostaglandins relax smooth muscle of ductus arteriosis.

Treat HUS by reducing BP, treating hyperkalemia, and transfuse for severe anemia.   Transfuse in HUS with HGB<6.   Restrict or cautious  IV fluids and consult Nephrology for dialysis.  HUS is most common cause of renal failure in kids.  Most common trigger is Ecoli toxin.   Ask for missing vital signs on the oral boards. 

Treat INH overdose with benzos, phenobarb, and pyridoxine.   Talk to family members or EMS on the oral boards.  They have key info.  INH interferes with pyridoxine which is needed to synthesize GABA.  Gyrometra muschroom (False Morel) overdose acts the same way as an INH overdose.  

If giving TPA for stroke, no heparin/asa/plavix.  If a patient is on heparin, lovenox, asa, plavix then you can’t give TPA.

AKA can develop hypoglycemia.  AKA is a starvation ketoacidosis due to emesis/gastritis.  Treat AKA with IV hydration with glucose containing fluids and give thiamine.  In aspiration setting, lateral decubitus films are classically described.  The dependent hemithorax(down side) should compress.  If it remains expanded then you need to suspect bronchial obstruction.

The lateral cspine view may be helpful in the trauma patient in shock to get quick eval for spinal fx and neurogenic shock.

Electrical Alternans is a sign of Pericardial Effusion/Tamponade. 

Abhi  Khatiyar  ORTHO STUDY GUIDE

Patellar dislocation mostly occurs in lateral direction.  Reduce patella and use knee immobiizer. 

Knee dislocation, beware popliteal vascular injury.  Examine for hard and soft signs of vascular injury. ABI<0.9 is abnormal.  All knee dislocations need angiogram. 

Tibial Plateau fx is most common fx of knee.   Lateral plateau is most common.   Beware in the older patient who has subtle or negative xray and can’t bear weight.

If you see fat globules in synovial fluid it strongly suggests fx.

Gout crystals are negatively birefringent.  Pseudogout (calcium pyrophosphate deposition disease) crystals are positively birefringent.

Thompson’s test evaluates for Achilles’ tendon rupture.   Positive test means the forefoot doesn’t move with calf compression in prone/flexed knee position. Fluoroqiunalones can increase risk of rupture.

Adhesive capsulitis (frozen shoulder).  Common complication after shoulder fx.  Can occur after stroke as well.

Carpal Tunnel Syndome: most specific sign is splitting of sensation of the 4th finger.

Lover’s fx (Don Juan Fx)is a calcaneal fx from fall from height.  Associated injury is burst fx of lumbar spine.

Jones fx of 5th mt is a  diaphyseal fx, slow to heal and has high incidence of non union.   Pseudo Jones Fx is an  avulsion fx’s off of the proximal 5th mt.

ROBBIE McDERMOTT   RADIATION MEDICINE PART 2

 Tx for local radiation injury: infection control, wound care, surgical consult, nsaid’s, hyperbaric oxygen, trental, vitamin E, topical steroids.

Acute Radiation Syndrome can occur from external exposures >1Gy.  DNA damage to cells within microseconds.  Intestinal cells, lymphocytes and stem cells are most prone to injury.

Stages of ARS: prodromal/nausea and vomit, fever, conjunctivitis; latent period; illness onset; recovery or death.

Hematopoietic syndrome(2Gy)   Lymphocytes and marrow cells are most sensitive to radation.

Gastrointestinal Syndrome (6Gy)  Earlier emesis=higher exposure.  If pt vomiting in 10 minutes or less than 60 minutes=bad outcome.

Cardio/CNS syndrome (20Gy)  Not usually survivable.

2Gy and less exposure has almost 100% survival.

Cytogenic Biodosimetry=# of dicentric chromosomes gives best estimate of radiation dose.

Absolute lymphocyte count is best test in first 24 hours for estimating radiation exposure.

Irradiated patients generally do not pose a threat to care providers.  Treat medical/surgical patients first.

Time to emesis if less than 2 hours likely exposure of 3Gy and greater.   Be aware that psychogenic emesis is common with radiation event.

Triage score=N/L+E.    E=0 for no emesis,   E=2 for emesis.   N/L is neutrophill/lymphocyte ratio.  Nl =2.21  If T>3.7 radiation dose is high.

 Colony stimulating factors for exposures >3Gy.   Implement  IDSA guidelines for neutropenia.

Amylase is another marker (baseling and 24 hours) for radiatin exposure.  Amylase will increase with significant exposure.

Contaminated patients in ER need to be kept in strict isolation.  Remove patient clothing.  Wash off patient and save all fluid runoff in a closed system.  Don’t let wash fluid run down drain.

Exposed patients with no symptoms for 6-8 hours can be discharged.

ER caregivers  for contaminated patients  should wear a  whole body coverall, surgical mask, double glove.

DAVE BAROUNIS   FEIBA FOR REVERSAL OF Coumadin COAGULOPATHY

Life threatening bleeding and INR>5 use 1000U of FEIBA over 15 min plus 10mg of IV vitamin K over 30 minutes

Life threatening bleeding and INR<5 use 500U of FEIBA over 15 min plus 10mg of IV vitamin K over 30 minutes

INR should be repeated in 30 minutes.   If INR<5 then you are done.   If INR still>5 give another 500U.

JOELLEN CHANNON   M and M   Massive PE

 Massive PE: sbp <90 for 15 minutes.   Submassive PE: RV dilation on echo.  Low risk PE has neither.

Mortality is highest with massive pe and lowest with low risk pe.

McConnell’s sign: apical contraction with wall motion abnormalities away from apex.

On CT if RV diameter/LV diameter >0.9 is sign of RV dysfunction.

Elevated troponin in PEis indicative of increased short term mortality.

T wave inversion inferiorly and anteriorly is a sign of RV strain.

TPA for submassive PE may result in better RV pressures at 6 months.   No one has been able to shows mortality benefit so far in this group.

Ways to identify the sicker patient with pe: echo, shock index>1, any hypotension at all, any respiratory distress.  These patients should be strongly considered for TPA.

If high probability of PE give heparin during workup; Recommended by AHA

 No TPA for undifferentiated arrest.

Effect of respiratory depression with benzos is pronounced in patients who have some aspect of co2 retention.

LOUIS HERRMANN  PEDIATRIC HIP PAIN

Legg-Calve-Perthes (idiopathic osteonecrosis/avascular necrosis)more common in short statured male  kids.  Insidious in onset.  Lesion on xray is more common on lateral aspect of femoral head.  Conservative management usually works well

If patient has thigh or knee or groin pain gotta evaluate the hip.

SCFE  more common in 11-14 years of age.  More common in Obese kids.  Related to thyroid disease.  Kids prefer his in abduction with external rotation.   Internal rotation is painful and limited due to the altered mechanics of hip.   Klein’s line runs along lateral femoral neck and should hit the epiphysis.  Also on xray lesser trochanter will be more prominent on xray due to external rotation.   SCFE requires surgery.

Transient synovitis: most common cause of hip pain in children.  Post viral process is thought to be the cause.   Non-toxic appearing patient. May have some fever. 

Septic Arthritis: More prominent effusion compared with transient synovitis.  Can be a tense effusion.  ROM of hip is commonly exquisitely tender.  U/S can identify the effusion.   Synovial fluid with wbc count>50000 is diagnostic for septic joint.   Surgical treatment with arthrotomy and joint wash out with 2 weeks of iv antibiotics is the treatment.   4 factors are more predictive:fever, non weight bearing, elevated esr >40, wbc>12,000.

 

 

  

 

Conference Notes 3-6-2012

Conference Notes  3-6-2012

GOTTESMAN-WATTS ORAL BOARDS

Case 1:Brugada Syndrome.  More common in Asian males.  IRBBB with septal ST elevation.  Fever can elicit EKG changes.   Structurally normal heart.  It is an electrical problem.  Pt’s need an AICD

Case2: Aortic Dissection     Treated Pain well.  Thorough/Efficient  ROS and PMH.  Should give labetalol or some combination to decrease pulse pressure.  If you give a combo, give the betablocker first.  Ntg alone not best choice.  Nitroprusside is better choice with a beta-blocker .

Case3:  Flexor tenosynovitis.  Treated pain well.  Gave Rocephin.  Ring on adjacent finger should be removed.  Staph is most common etiology.  Kanavel’s sings, fusiform swelling, pain with passive

Gupta  Study Guide Trauma

Tension pneumothorax may have equal breath sounds.

CT abdomen is superior to ultrasound for diagnosing solid organ injury.

Indications for CT  for diagnosing renal injury is gross hematuria, hemodynamic instability, rapid decal injury.   Textbooks recommend In kids consider ct for microscopic hematuria.  U/S is the better test in kids in 2012 to reduce radiation.

ED thoracotomy is most likely to save a patient with cardiac tamponade due to a stab wound to the heart.

Myocardial contusion is a diagnosis that you don’t need to worry about unless the patient is hemodynamically unstable or has a significant arrhythmia.

Abdominal wall ecchymoses should raise suspicion of duodenal hematoma.  Initial CT may be neg.  Pt needs serial exams if they have continued pain.

Extraperitoneal bladder ruptures usually occur from lacerations due to pelvic fx.

LFT’s are not useful in trauma.  Pancreatic injuries may initially have normal amylase and lipase.

Most commonly injured organs in penetrating abdominal trauma:  small bowel>liver> colon.

Most commonly injured organ in blunt abdominal trauma: spleen

Treat rib fractures/contusions with pain medication and incentive spirometry. 

Left sided diaphragmatic injuries are 3X more common than right sided.   Delayed herniation is possible. These injuries do not heal spontaneously.   Best diagnostic test is laparoscopy.

Hemodynamically unstable patients due to pelvic fracture: tie pelvis with bed sheet,  next move is angiography with embolization.

EASTVOLD TOMASELLO  

ANTERIOR ISCHEMIA

Early repol: diffuse st elevation, nl r wave progression, short qt interval, nl t wave morphology, no reciprocal changes , J point notching

More likely a STEMI vs Bening Early Repol if greater ST elevation, longer QT (QTCmore than 392), lower R wave amplitude in V4 (less than 13mm).

If T Wave exceeds QRS amplitude, it suggests STEMI.

Pericarditis:  PR depression is most prominent in V5 and 6.   You will never have ST elevation with pericarditis in lead V1.   Check AVR for PR elevation.

Hyperkalemic T waves are pointed at their most superior aspect and there are flat ST segments.

Wellen’s Syndrome: Biphasic t waves leads V2-3.  Suggestive of proximal LAD occlusion.   EKG findings are seen when pt is pain free.

De Winter’s Sign:  Anterior ST depression followed by very prominent T waves.  Associated with proximal LAD occlusion.

ST SEGMENT CHANGES

  Flattened ST segment is fairly specific for ischemia

ST segment change sconcordant with the QRS  are indicative for ischemia

Subendocardial ischemia does not localize, it is diffuse.  So if you see focal st segment depression, look hard for ST segment elevation somewhere else.

Infero-lat ST depression with STE in AVR=severe left main dz or 3 vessel DZ.

If you see inferior st depression, look for ST elevation in the high lateral leads.

Millimeter criteria for STEMI will often fail you.  Look for patterns.

Localized depressions V1-3, think posterior ami.   If there is diffuse st segment depression it is subendocardial ischemia.

Isolated t wave inversion in AVL, think ensuing inferior MI.

Serial EKG’s are critical!

If ST elevation is greater in lead 3 than lead 2 in an inferior MI, that is suggestive of RV infarct as well.

Posterior MI has anterior ST depression with flat ST segments, upright t wave, and tall R waves.

Abnormal T wave balance: T wave in V1 is larger than V6 or 1.  Bundle branch blocks and LVH are the two times when this rule doesn’t work.

Pericarditis: AVR shows PR elevation and st depression,  V1 should have no pr or st changes.

RV strain=RBBB, tachycardia, T wave inversion in 3 and V3, terminal r wave in V1, S1Q3T3, 1 and AVL with upright T waves.

TCA overdose look for large terminal R wave in AVR.  Benadryl and cocaine can do the same thing.

PAILS: mnemonic for reciprocal changes  Post-Ant, Inf-Lat, Lat-Septal

 

Conference Notes 2-21-2012

Conference Notes 2-21-2012

TAKING YOUR PRESENTATION TO THE NEXT LEVEL WORK SHOP

 

CARLSON   TOXICOLOGY 3  STUDY GUIDE

Decon rat poison does not need active monitoring or treatment if a toddler has a single ingestion.   For an intentional ingestion for suicidal ideation these patients require close monitoring of INR and possibly vitamin K over several days.

Methyl chloride ingestion cause carbon monoxide poisoning

If iron ingestion has no symptoms for 6 hours it will not be a toxic ingestion.

Macrolides can prolong the QT interval resulting in torasades.  

Excessive vitamin C intake can cause kidney stones.

Methemoglobinemia is commonly due to benzocaine, or pyridium.  Pulse ox will be around 85%.   Pt will be cyanotic.  Treat with methylene blue.

Button batteries >15mm are at increased risk of impaction in the GI tract.  Button battery in the nose or esophagus needs to be removed immediately.

ORAL BOARDS   COGHLIN AND KUTKA

Case 1.  Bronchiolitis requiring intubation.  Had to trouble shoot hypoxia following intubation due to oxygen tubing becoming unplugged.

Case 2.  Guillain Barre Syndrome.   Check vital capacity.  Give IVIG.  ICU admit with neuro on board

Case 3.  CRAO.   Occular massage, breathe in paper bag, give timoptic an acetazolamide.  Consult Ophtho

RADIATION EXPOSURE MEDICINE   MCDERMOTT

Alpha particles: 2 protons/2 neutrons.  Can be blocked by paper/wall/skin.  Inhaled particles are bad.  Uranium, plutonium are sources.

 

Beta particles: Electrons.  blocked by plastic sheet.   Tritium, Strontium are sources

Gama Rays: Can create ionization.  Higher energy than xray. Easily travels thru matter.  Blocked by lead or thick concrete.  Can hit bone marrow, gi tract, brain.

Neutrons: Uncharged particles. Can cause another material to be radioactive.

Irradiation=exposure with no material transfer.  Contamination=material transfer.

Basic unit of radioactivity is the Curie.  1 Gray=1 sievert.

Important isotopes to know: Iradium, Cesium, Cobalt (industrial 3).   Hydrogen, Uranium, Plutonium, Americum (military 5 hard to get and enrich).

Geiger Counter measures radiation present from contamination of an area or patient.

Personal protection is fairly effective with just coveralls, gloves, and face shields. 

Decontamination of patients is best in a closed space in an isolated area of ED.

Conference Notes 2-14-2012

Conference Notes 2-14-2012

IN TRAINING ZEBRAS    GIRZADAS

 

STEMI CONFERENCE

Out of hospital arrests are contributing to high post- PCI mortality rate.  If patient has prolonged resuscitation, decision to go to cath lab needs to be discussed with cardiologist.

Posterior-Lateral ST segment elevation suggests circumflex artery lesion.

Cardiogenic shock after cardiac arrest is a strong marker for eventual death.

Concordant st elevation with a bundle branch block is very suspicious for AMI.   Similarly, “ironing out” or straightening of the st segment is also very suspicious for an impending stemi. 

You shouldn’t see a Q wave in lead 1 in a LBBB.

DOCUMENTATION   McGURK

Document that you visualized and interpreted  the imaging study you ordered.  

TRAUMA GSW TO TORSO NON-OPERATIVE MANAGEMENT  PATEL

Pt has to have hemodynamic stability and no peritonitis to be considered for non operative management.   Also cannot have signs of hollow viscous injury on CT.

Serial abdominal exams are required.  This has the best sensitivity and negative predictive value for intra-abdominal injury.   Serial lab testing is also required.

Laparascopy is used for penetrating LUQ injuries to look for diaphragmatic injuries.

Some Trauma research has shown success with non-operative management of penetrating torso trauma.

FAST exam has little utility for penetrating abdominal trauma.  It is too nonspecific.  Blood could be coming from solid organ injury.  It is useful in penetrating trauma to the chest to rule out pericardial tamponade.  

Negative lap carries a lifetime risk of 15% for SBO.

CASE F/U  AFIB WITH RVR  COLLANDER

Afib with RVR: 60% convert with 100J biphasic.   80% with 200J biphasic. 200 J is a reasonable starting dose of electricity.   Use synchronized cardioversion.

Predisposiing factors for Afib: rheumatic heart, htn, etoh binge drinking, ischemic heart disease, obesity.  Fish oil may decrease risk.

Who should be cardioverted?  Unstable, first episode, infrequent episodes, worsening symptoms.

Who should not be converted?  Asymptomatic, elderly with multiple comorbidities, bleeding risk, symptoms more than 48 hours.

If you get them back into sinus, they can go home without meds.

Ottawa Protocol:  1 gram of Procainamide over 1 hour first,  if not successful cardioversion with 200J.

No need for anticoagulation following successful cardioversion.

If you fail with cardioversion, you can re-try with pt in exhalation, or use disconnected paddle to press zoll pad down more firmly.

Elise's conference pearls from 1-17 (also sent via email)

From Study Guide:
1.  Neutropenic fever and rectal exams:  7th Edition Tintinalli does say digital rectal exam is relatively contraindicated in neutropenic patients, and should be withheld until after antibiotics are started.  It also says to pay attention to the oral exam, perianal exam and entry sites of IV catheters; areas of infection not commonly evaluated in non-neutropenic patients.
2.  Coagulopathy and paracentesis:  7th Edition Tintinalli also says to reverse coagulopathy and thrombocytopenia before doing paracentesis, so correct answer for the test, but probably not the correct answer in real life:
Hepatology. 2004 Aug;40(2):484-8.

Performance standards for therapeutic abdominal paracentesis.

Source

Advanced Liver Diseases Study Group, Division of Gastroenterology and Hepatology and Internal Medicine, Mayo Clinic, Rochester, MN 55905, USA.

Abstract

Large-volume paracentesis, the preferred treatment for patients with symptomatic tense ascites due to cirrhosis, has traditionally been performed by physicians as an inpatient procedure. Our objectives were to determine (1) whether large-volume paracentesis could be performed safely and effectively by gastrointestinal endoscopy assistants and as an outpatient procedure, (2) whether the risk of bleeding was associated with either thrombocytopenia or prolongation of the prothrombin time, and (3) the resources used for large-volume paracentesis. Gastrointestinal endoscopy assistants performed 1,100 large-volume paracenteses in 628 patients, 513 of whom had cirrhosis of the liver. The preprocedure mean international normalized ratio for prothrombin time was 1.7 +/- 0.46 (range, 0.9-8.7; interquartile range, 1.4-2.2), and the mean platelet count was 50.4 x 10(3)/microL, (range, 19 x 10(3)/microL - 341 x 10(3)/microL; interquartile range, 42-56 x 10(3)/microL). Performance of 3 to 7 supervised paracenteses was required before competence was achieved. There were no significant procedure-related complications, even in patients with marked thrombocytopenia or prolongation in the prothrombin time. The mean duration of large-volume paracentesis was 97 +/- 24 minutes, and the mean volume of ascitic fluid removed was 8.7 +/- 2.8 L. In conclusion, large-volume paracentesis can be performed safely as an outpatient procedure by trained gastrointestinal endoscopy assistants. Ten supervised paracenteses would be optimal for training the operators carrying out the procedure. The practice guideline of the American Association for the Study of Liver Diseases which states that routine correction of prolonged prothrombin time or thrombocytopenia is not required is appropriate when experienced personnel carry out paracentesis.

____________________________________________________________
3.  From GI Curbside Consult:  IV erythromcyin now well accepted pre-endoscopy, and probably better than NG in cleaning out upper GI tract to help with visualization:
Aliment Pharmacol Ther. 2011 Jul;34(2):166-71. doi: 10.1111/j.1365-2036.2011.04708.x. Epub 2011 May 25.

Meta-analysis: erythromycin before endoscopy for acute upper gastrointestinal bleeding.

Source

Department of Gastroenterology, Changhai Hospital, Second Military Medical University, Shanghai, China.

Abstract

BACKGROUND:

Studies evaluating the effect of erythromycin on patients with acute upper gastrointestinal bleeding (UGIB) had been reported, but the results were inconclusive.

AIMS:

To compare erythromycin with control in patients with acute UGIB by performing a meta-analysis.

METHODS:

Electronic databases including PubMed, EMBASE and the Cochrane Library, Science Citation Index, were searched to find relevant randomised controlled trials (RCTs). Two reviewers independently identified relevant trials evaluating the effect of erythromycin on patients with acute UGIB. Outcome measures were the incidence of empty stomach, need for second endoscopy, blood transfusion, length of hospital stay, endoscopic procedure time and mortality.

RESULTS:

Four RCTs including 335 patients were identified. Meta-analysis demonstrated the incidence of empty stomach was significantly increased in patients receiving erythromycin (active group 69%, control group 37%, P<0.00001). The need for second endoscopy, amount of blood transfusion and the length of hospital stay were also significantly reduced (all P<0.05). A trend for shorter endoscopic procedure time and decreased mortality rate was observed.

CONCLUSIONS:

Prophylactic erythromycin is useful for patients with upper gastrointestinal bleeding to decrease the amount of blood in the stomach and reduce the need for second endoscopy, amount of blood transfusion. It may shorten the length of hospital stay, but its effects on mortality need further larger trials to be confirmed.

 

4.  From Joint Peds/EM conference:  This is just a reiteration of an excellent point made during the discussion:  for healthy, self-limited new onset seizure in peds patient, NO emergency neuro-imaging needed unless: focal neuro deficit, prolonged altered state, fever, or focal seizure.  If the kid needs an emergent neuro-imaging study due to one of these reasons, MRI far preferable.  All kids will get EEG, try to arrange within 24 hours for improved predictive value, and EEG results will guide need for outpatient MRI.

Conference Notes 1-17-2012

Conference Notes 1-17-2012

JOINT  EM-PEDIATRIC CONFERENCE     SEIZING CHILD

Simple Febrile Seizure:  6mo to 6 years, generalized tonic clonic sz lasting less than 15 min, no focality, pt has temp >38.     Consider blood glucose level.   LP is not mandatory.  Approach child as having no seizure at all and look for source of infection with history and physical exam.   Don’t get a CT brain on these kids.   Admit if child has more than 1 sz in 24 hours or any sign of complex febrile seizure.  If seizure does not occur in first day of illness this could be a red flag and consider admission.  Similarly, if first febrile seizure occurs after the age of 5 the pt should have a neuro eval inpatient or outpatient. 

 

First Non-Febrile Seizure:  Careful clinical exam for any focal neuro deficit.  Labs should be ordered based on individual clinical circumstances.   UTox should also be considered.  EKG should  be done only with a history of syncope.  No EKG needed if seizure is focal.   LP only with concern about meningitis.  EEG is a recommended outpt option in all these kids.   EEG within 24 hours is best if possible but don’t go nuts trying to get this arranged from the ER.  Focal seizures, Todd’s paralysis, not returning to baseline within a few hours  all require an ED CT. If you can get the radiologist to do the MRI do that instead of CT.  Kids with first time generalized seizure with no focality, no Todd’s paralysis, and no  failure to return to baseline do not need ED imaging.  Seizures in kids who are on seizure meds don’t need imaging even if seizures have some change in character from usual or child has trivial head injury. If child returns to normal after seizure they can go home and get outpt work up.   Admit kids with first time seizure who are under 6mo.  

 Status Epilepticus:  First line is Benzos,  Second line: Fosphenytoin.    Can give phenobarb 5-10mg/kg prior to getting level if patient is on phenobarb at home.    Third line: IV Keppra,   IV Depacon.    Fourth line: Propofol.     If kids get to the ED per EMS or family still seizing that in effect is status.    Loading levels for all seizure meds Is 20 mg/kg.

Sudden Unexplained Death in Epilepsy:  More likely in patients whose seizures are poorly controlled.   Adults more common than kids.    Adults:1:650,   Kids 0.2:1000

 

HAYWARD   HEME-ONC  STUDY GUIDE

RDW changes prior to MCV in deficiency anemias.

Platelet deficiency usually shows up as petechiae, epistaxis, mucosal bleeding, but not deep tissue bleeding.

D-Dimer can be normal in liver disease but should be very abnormal in DIC.  Thus it can differentiate between the two. 

Low platelets is the most common lab abnormality in DIC.

Most common cause of DIC is sepsis

In hemophilia give factor 8 prior to getting imaging study done.   50 international uinits for possible head bleeds (100% of factor activity),  and 25 IU for joint bleeds (50% of factor activity).

DDAVP is mainstay of treatment for kids with type 1 von Willebrand Disease.

Hemolysis due to transfusion reaction will result in decreased serum haptoglobin, increase ldh, increased serum hemoglobin and hemoglobinuria.   Increased LDH and low haptoglobin are 90% specific for hemolysis.

TTP= thrombocytopenia, MAHA, fever, renal impairment, and neurologic impairment.

Treatment of TTP is plasma exchange.

GROMIS-RADAWI  CURBSIDE CONSULT GI BLEED

Black stool means digested stool. It passed through small bowel.   90% Started above the ligament of treitz.    10% can come from right colon.   Melanotic  is an incorrect term.  Melenic is the correct term.  Hematochezia is 90% colonic bleed but 10% have upper gi source.   That 10% is at high risk of shock from an arterial bleed in an ulcer.

Medications in GI bleed are secondary to identification of major bleeder and circulatory support.   Use of PPI’s in GI bleed started from data showing that blood clotted better in stomach when ph was higher.  Most GI bleeders don’t need continuous PPI drip.  Major league bleeders should probably be on continuous drip.

NG tube can be helpful to assess the pace of bleeding.   There are false negatives however.  Helps to better visualize GI tract with endoscopy.   Varicies do not contra-indicate NG tube placement. 

IV Erythro can be used to induce gastric motility and improve the endoscopic visualization of the stomach and small bowel.

ANDREA CARLSON    POISONING WITH CV DRUGS

Class 1a: Quinidine is prototype.   Procainamide is more common.    Infusions can result in hypotension and QRS widening.    Disopyramide is a strong negative inotrope.  Blocks pancreatic islet cells resulting in hypoglycemia.    QRS widening/QTC prolongation due to sodium channel blockade.    Tx with Bicarb for QRS widening.   Avoid acidosis.  Prolonged resuscitation is indicated.   You can try lidocaine (1b).

Class 1C: Flecainide and Propafenone.   Strong NA channel  blockers.  Effects are hypotension, bradycardia,  QRS/QT prolongation.  Flecainide can cause Brugada Pattern.   Tx with Bicarb.  For propafenone follow up with hypertonic saline.  Amiodarone (Class 3), lipid rescue, and pacing may help.

 

Class 1A/1C Mimics: TCA’s, cocaine, phenothiazines, Benadryl, tegretol, choloral hydrate, propoxyphene.

Class 1b:  Lidocaine and Mexiletine.    Also NA channel blockers but selective for rapidly depolarizing or ischemic cells.   At times used as a cutting agent for cocaine.  Can cause methemoglobinemia.    Fasiculaitons and seizures can result from bigger OD’s.  Treat seizures with benzos or phenobarb.   Don’t use phenytoin because it is a 1b also. 

Class 3: Amiodarone acts at the potassium channel.   Sotalol is a potassium channel blocker and beta blocker.   Tx is supportive for these.   Glucagon may benefit sotalol OD.   Multaq (Dronedarone) is a new Class 3 drug but has a lot other properties (dirty drug).

Adenosine is unclassified drug.   Increases AV nodal refractoriness.   No reports of overdose.  Use lower dose in cardiac transplant patients and those using persantine.     Increase dose in patients on methylxanthines.    Caution in asthmatic patients theoretically could induce bronchospasm.

BILL SCHROEDER    CONCUSSION/HEAD INJURY EVALUATION

SCAT2 :   Ask the head-injured player questions about this game and the last game.    Do the standing tandem gait balance test for 20 seconds.   More than 5 adjustments in that 20 seconds requires pulling the kid from the game.

After head injury, kids need rest with no physical activity or school until symptoms resolved without Tylenol/ibuprofen.    Limit video games and texts.    80-90% of concussions will resolve in 7-10 days.

Post-traumatic headaches can be treated with amitriptyline.  Topomax if there is a history of migraines.

Sertraline helps with cognitive effects following head injury.     Other medications possible are Ritalin and Concerta.

Step wise progression for return to play.  Don’t even start until symptom free.   As the patient increases activity, if symptoms recur they need to rest for 24 hours and restart at lower level.

Children should not return to play the same day as concussion because they are more prone to cerebral edema in “second impact syndrome”.   Only 2 cases in adults.   Seen basically only in teenagers.   Rapid death 2-5 minutes after second impact.  Peak incidence of second impact syndrome in the mouse model is 3 days out from first injury.

Concussion is due to stretching of neuronal and axonal membranes. Ion influx then other bad oxidative and metabolic stuff in the nerve cell.

RESIDENT SELECTION COMMITTEE MEETING

 

Conference Notes 1-10-2012

Conference Notes 1-10-2012 (sorry but they are abreviated due to lecture/admin responsibilities this day)

BAROUNIS/KONICKI    ORAL BOARDS

Case 1: Hypothermia requiring active rewarming with cardiopulmonary bypass or cool guard catheter.   The patient needed a low measuring bladder thermometer to accurately identify the patient’s temperature.

Case 2:Intussusception

Case 3: Perilunate dislocation

JIM MALETICH   VERTIGO

MAGGIE PUTMAN  /NICK KETTANEH   5 MIN F/U’S

GIRZADAS   NURO STUDY GUIDE

ALYSSA   AV  BLOCKS

Lyme disease can cause third degree heart block.

3rd degree is defined by AV dissociation.

Weinkebach is 1st degree type 2.  PR progressively lengthens and then a qrs is dropped.  The RR interval shortens as the pr lengthens.

INNAUGURAL EM-IM CASE  CONFERENCE

Patient had critical Upper GI bleed due to varicies.

NG tube is not contraindicated by known varicies.

Give blood, octreotide,protonix,antibiotics,  iv fluids, reverse INR  with ffp/vitamin k if increased.

80+% of  upper GI bleeds will be due to gastritis or pud.   If you suspect varicieal bleeding (the minority) based on physical exam or lab testing, start octreotide.

Variceal bleeders with upper GI bleed have a 50% mortality.

Cirrhotics have a baseline hyperdynamic cardiovascular state.  They may have low bp at baseline due to being chronically vasodilated.

Chronic treatment with any beta blocker in a patient with varicies lowers portal pressure and decreases chance of rebleeding of varicies.   The target heart rate is bradycardia in these patients.   Cipro is used prophylactically to decrease risk of SBP.    Cipro does not worsen liver disease induced coagulopathy like it does for warfarin induced coagulopathy.

Give variceal bleed patients prophylactic antibiotics.  It has been shown to decrease rebleeding an mortality.

Intubate these patients with ramped or head up position.   Pre-oxygenation is critical.   Use a neuromuscular blocker to optimize your view.    You can also try using a meconium aspirator to improve your ability to suction.    LMA can be a bridge device.

In coding cirrhotic patient be concerned about hypokalemia and hypomagnesemia.